Download as docx, pdf, or txt
Download as docx, pdf, or txt
You are on page 1of 98

Part One: Reading

1- Read the following article and then answer the questions that follow.

A. According to the old proverb, “the early bird catches the worm,” this is certainly
true when it comes to deciding your future. You will get off the best start with your
job- hunting if you think about it while you still at school. Does your school have
careers officers? If so, you should take advantage of them, as this will give you an
idea of what's on offer and help you decide what job will suit you. Remember! Finding
out as much as possible will help you make a better-informed decision.

B. After you have left school, you may think about applying for any jobs that you are
qualified to do. But, think about the long term. Does this job have future prospects? Is
it dead-end or does it have chances of promotion? Will you be happy doing this job
in five, ten- or twenty-years’ time? To open the door to a wider variety of jobs, focus
on further education and training.

C. You should also take yourself into account. What are you good at? What are you
interested in? Are you a 'people person' or happiest with your own company? Do you
have a creative flair or are your adept at arithmetic? Don’t just apply for a job because
it has a good salary. It is important, of course, but ask yourself about what can the job
offer you in terms of personal satisfaction?

D. Be systematic in your search. First, write down possible career paths then think of
the different jobs within this field. Think of the skills and qualifications you need to
get that first valuable job, and how you might go about getting them. Don’t just limit
your thinking to further study. There may be other, less immediately obvious routes,
such as doing voluntary work.

E. Traditionally, a good source of job advertisements has been newspapers, job


centres and word of mouth (friendly and family can have the most useful information
of all). But nowadays, any job search would not be complete without access to the
Internet.
A) Match the paragraphs (A – E) with the suitable headings. Note that there is one extra heading.
Heading Paragraph
Think about yourself
Where to look for a job
Start looking early
How to write your CV
Consider the future
How to look for a job
B) Answer the following questions. Write short answers.
1. What is the best time to look for a job?
……………………………………………………………………………………………………………………………………………………………
2. How does a school careers officer help student?
……………………………………………………………………………………………………………………………………………………………
3. What does the underlined pronoun (them paragraph A) refer to?
……………………………………………………………………………………………………………………………………………………………
4. What does the underlined pronoun (It) in paragraph C refer to?
……………………………………………………………………………………………………………………………………………………………
5. What is the most important source to get a job nowadays?
……………………………………………………………………………………………………………………………………………………………
C) Match the words to the correct definition: (There is one extra definition)
proverb – promotion – offer – obvious - Traditionally

Word Definition
the act of raising someone to a higher or more important position or rank.
now or without waiting or thinking.
according to tradition; in a traditional way.
easy to see, recognize, or understand.
a short statement, usually known by many people for a long time, that gives advice
or expresses some common truth.
to provide or supply something.
D) Read the following sentences and decide whether they are true, false or not given by putting
tick (√) in the correct place:
The sentence True False Not Given
1- Early thinking of a job helps you find good one.
2- To feel happy at work is not important.
3- You have to be so confident during job interview.
4- The internet plays an important role in finding a job nowadays.
2- Read the following article about the changing environment of Madagascar and then
answer the questions that follow.

1- Madagascar is the fourth largest island in the world. It is located in the Indian Ocean off
the southeastern coast of Africa. The country is quite poor and most people earn less than
$2 a day. However, there is growing interest in the country for tourism, but at the moment
very few tourists visit.
2- Madagascar has something special which makes it very interesting for scientists and
tourists alike. It has many species of plants and animals that do not exist in other countries.
Between 80 and 90 percent of its animals and plants have only ever been found in
Madagascar.
3- People have lived in Madagascar for a very long time. Most researchers believe that the
first people came to the island over 2000 years ago from Borneo in the South China Sea.
However, since then Arabs and East Africans, and later Malays, Indians, Chinese and Europeans
have all lived in Madagascar. The population has risen to over 20 million people and this has
caused many environmental problems.
4- Nearly fifty percent of Madagascar's forests have disappeared in the last fifty years. It is
even higher on the coast. Scientists have estimated that nearly 50 percent of Madagascar's
animals and plant species are dead or will die. Madagascar's most famous animal is the
lemur. Unfortunately, at least 17 species of lemur have become extinct since people arrived.
Many plants and animals we know little about will be dead before we have a chance to
know more about them.
A) Read the following sentences and decide whether they are true, false or not given by putting tick
(√) in the correct place:
The sentence True False
1. Lots of people go on holiday to Madagascar.
2. Many plants and animals are unique to Madagascar.
3. Madagascar's population is not causing problems for the island.
4. The forest damage is worst on the coast.
5. Some species of lemur are in danger.
B) Match the following words from the article with their definitions. Note that there is one extra
definition earn – interest – exist – cause- estimate
Word Definition
having no living members
get money by working.
the feeling of wanting to know or learn about something.
to guess the amount of something.
to live in difficult conditions.
make something (especially bad) happen
3- Read the following article and then answer the questions that follow.

(1) The Kingdom of Bahrain has many different attractions to please tourists and
visitors. For history lovers, there is the rich heritage of ancient civilizations, which date
back to over 5,000 years. A good place to start with is the Bahrain National Museum
where you can learn about Bahrain’s rich history. In addition, a tourist can go to the
Pearl Museum and learn about the history of pearl diving in Bahrain and how
hardworking the old Bahraini people were.
(2) Many tourists from the West come to Bahrain in winter to enjoy the fantastic mild
climate. They enjoy all kinds of water sports such as diving, swimming, sailing and
water rafting. They find the beaches of Bahrain the best places to spend winter in
owing to the warm nice clean water and the large number of facilities they have.
(3) Forts at Riffa, Arad and, of course, the Bahrain Fort itself are remarkable examples
of old architecture. Closer to the present, historically speaking, you can visit the
restored houses of Muharraq which represent Bahraini style of homes as they were
during the last years. They show how clever the Bahraini builders were and how
simple and pure life in the past was.

(4) Nature lovers may visit the Al Areen Wildlife Park where they will see flamingos,
ostriches, zebras and camels or take a trip to the centre of the Island and gaze in
wonder at the Tree of Life which has survived for hundreds of years in the middle of
the desert.
(5) For modern-style shopping, there are so many big shopping malls all over
Bahrain. Some of the biggest malls are, Al A’ali Shopping Complex, Seef Mall, Bahrain
Mall, Dana Mall, and Country Mall. These malls have everything shoppers may dream
of for a competitive price.
A) Match the following headings with the suitable paragraphs (1-5). Note that there is one
extra heading.
Paragraphs Headings
Winter in Bahrain
Old buildings in Bahrain
Well-known stores
Health care in Bahrain
History of Bahrain
Birds and animals
B) Match the following words from the article with their definitions. Note that there is one
extra definition
different – Museum – climate – remarkable - survived

Word Definition
a building in which old and historical objects are stored and exhibited
quick to understand
the weather conditions in general
noticeable; worthy of attention
continue to live or exist in spite of danger
various; more than one
C) Answer the following questions:
1- Why is Bahrain National Museum a good place to visit?
……………………………………………………………………………………………………………………………………………………………
……………………………………………………………………………………………………………………………………………………………
2- Mention THREE marine sports tourists can enjoy in Bahrain.
a) …………………………………… b) ……………………………………. c) ……………………………………
3- What do the restored houses of Muharraq show?
……………………………………………………………………………………………………………………………………………………………
……………………………………………………………………………………………………………………………………………………………
4- Name TWO birds you can see at Al Areen Wildlife Park.
a) ……………………………………………………………….. b) …………………………………………………………….
D) What do the highlighted words in the passage refer to?
Word Reference
The pronoun 'they' para.(2) line (4)
The pronoun 'They' para.(3) line (4)
The pronoun 'they' para.(4) line (1)
E) Find words in the passage with the SAME meaning
Word Synonym
entertain para.(1)
warm para.(2)
E) Find words in the passage with the OPPOSITE meaning
Word Antonym
modern para.(1)
hate para.(2)
farther para.(3)
4- Read the following article and then answer the questions that follow.

Action Adventures
At Action Adventures, we provide exciting holidays for sixteen- to twenty-one-year-olds.
Our centre is in the beautiful region of Cornwall in the south-west of England (about 350 km
from London).
Cornwall is famous for its dramatic coastline and long, sandy beaches. Our holidays are fun,
adventurous and safe, and there is the opportunity to meet other people and make new
friends. You can do a wide range of activities including water skiing, surfing, windsurfing,
sailing, climbing, and mountain biking. We have courses for all levels from beginner to
advanced. So, no previous experience of the activities is necessary.
The accommodation is fairly basic but comfortable. We have a choice of self-catering rooms,
caravans or camping. All the rooms have the necessary equipment for making snacks and
light meals. If you don’t want to cook in the evenings, you can always eat out at one of the
many restaurants nearby or come along to our evening barbecue.
If you are tired in the evening you can relax in our TV and recreation room. Alternatively, we
have a lively evening social programme with something for everybody seven nights a week.
You can watch and listen to live bands, go bowling or take part in our competitions, quizzes
and games.
Accommodation, lessons and equipment are all included in the price and there is also free
transport. The only thing not included is food.
A) Read the article and find the missing information.
1. Name two water sports:
a) ………………………………….. b) …………………………………….
2. Name two types of accommodation:
a) ………………………………….. b) …………………………………….
3. name two evening activities:
a) ……………………………………………………………………………………………
b) ……………………………………………………………………………………………
B) Read the following sentences and decide whether they are true, false or not given by
putting tick (√) in the correct place:
The sentence True False
1. Action Adventure holidays are for children.
2. It offers holidays for people with different levels of experience.
3. The accommodation hasn’t got any cooking facilities.
4. The evening activities are only at weekends.
5- Read the following article and answer the questions that follow.
Teen magazines
1- In the first part of this series, we saw that publishers do not make their money from the sales
of the magazine. They make it from the sales of advertising space. What effect does this fact
have on the appearance and contents of teen magazines for girls?
2- The teen magazine for girls is one of the most popular types in the western world. At its peak
in 1998, this type of magazine sold over 2.5 million copies a month in Britain. That is one for
every British female teenager at the time. Sales of teen magazines have fallen since then, but
this decline may be due to the rise in electronic magazines - or e-zines - on the Internet.
Teenagers are probably reading as much, but with different media.
3- So teen magazines and e-zines for girls are very popular. Researchers in media studies think
that they are also very influential. They think there is persuasion in these magazines. They
believe that they persuade their readers to buy certain products, live in a certain way and have
certain attitudes and values.
4- Some teen magazines reflect self-image, but some try to promote a particular kind of self-
image. You can do some research yourself. Buy a teen magazine and flip through it. Does the
magazine reflect your experience of being a teenager? Or does it try to persuade you to be a
certain kind of person? It may be saying: Teenagers should like these things, they should behave
this way and they should have those values.
5- Why do magazines try to persuade teenage girls to have certain values and a particular
lifestyle? Because they want to sell advertising space to companies with certain products
Answer the following questions.
1. The profit publishers make is related to the number of magazines sold. True/ False
2. Teen magazines are famous all over the world. True/ False
3. How many copies did teen magazine sell in 1998?
…………………………………………………………………………………………………………………………………………………
4. How many female teenagers were there in the UK in 1998?
…………………………………………………………………………………………………………………………………………………
5. Why have sales of teen magazines in the UK changed?
…………………………………………………………………………………………………………………………………………………
6. What do teen magazines try to sell to teenagers?
…………………………………………………………………………………………………………………………………………………
7. Find one word in the two first paragraphs that means ‘highest point’
…………………………………………………………………………………………………………………………………………………
8. Find one word in the last two paragraphs that means ‘having a strong effect’
…………………………………………………………………………………………………………………………………………………
9. The pronoun ‘they’ in the third paragraph refers to…
…………………………………………………………………………………………………………………………………………………
10.According to you, what is dangerous about the teen magazines. (write two negative effects.)
a. …………………………………………………………………………………………………………………………………………………
b. …………………………………………………………………………………………………………………………………………………
6- Read the following article and then answer the questions that follow accordingly.
1- Since he was 11, Babur has been running his own school in Bhabta, small village in West
Bengal, passing on to the children of poor families the knowledge he has acquired at his fee-
paying school during the day.
2- It began when children in his village plagued him with questions about what he learned at
the 1,000-rupee-a-year school their parents could not afford.
3- "It started without much effort." he says. "There were lots of children who had dropped out of
school, or never been to school at all. They were always asking questions about my lessons and I
would repeat everything for them.
4- "There used to be just eight pupils, and my friends helped me with the teaching. We worked
on the standard of teaching, the word spread to nearby villages, and gradually we took on
more and more students."
5- Five years later, Babur is recognised by district education officials as 'head teacher' of the
Anand Shikshya Niketan school, with 10 teachers and 650 pupils. The teachers work unpaid, the
children wear their own clothes rather than uniforms, and the books and desks are financed
through donations from rich people and charity associations.
6- Babur works remarkably long hours. On a typical day, he'll rise at 5 a.m. for morning prayers,
do household chores, then take a bus to school in a village three miles away. From 10am to
4pm, he focuses on his own studies, then he races back to his village to welcome his students at
5p.m. He teaches until 8p.m. and supervises his colleagues, mainly fellow pupils ranging from
16 to 19 years old. The teaching doesn't make him tired, he says, but gives him more strength to
keep up his busy schedule and manage the school business.
7- His parents are bursting with pride. His father, Mohammed Nasiruddin, was shocked when he
first discovered his son was teaching. "I couldn't believe it. He's always rushing from place to
place and I was worried it might affect his studies. So I visited his school. Seeing his
determination and dedication to teaching cleared my doubts," he says.
8- Babur believes he has found his vocation. He's working towards qualifying as a teacher so
that he can carry on developing his school. His plan is to sign up for a distance degree so that
he won't have to give up teaching classes.
9- The secret of his success, he says, is commitment. "You have to be dedicated and
determined. You need to create a positive learning environment. And there has to be goodwill
between teacher and students
A) Read the article and choose the correct answer a, b or c.
1. Babur’s school started with.............students only.
a) 8 b) 10 c) 650
2. The school is able to run without charging fees because:
a) district education officials promoted Babur.
b) students wear uniforms instead of clothes
c) some school items are donated
3. To Babur, teaching is a source of:
a) strength b) tiredness c) business
4. How do Babur’s mum and dad feel?
a) Shocked b) Worried c) Proud
5. Babur is planning to:
a) go on holiday b) continue his studies c) give up teaching
B) Match the following words from the article with their definitions. Note that there is one
extra definition.
gradually - donation - colleagues - vocation - commitment
Word Definition
People that you work with
A promise or firm decision to do something
The work that a person does.
To make someone suffer for a long time
Money or goods given to help a person or organization
Slowly over a period of time
C) What do the highlighted words in the passage refer to?
Word Reference
The pronoun 'his' para.(1) line (2)
The pronoun 'their' para.(2) line (2)
The pronoun 'it' para.(7) line (2)
D) Read the following sentences and decide whether they are true, false or not given by putting tick
(√) in the correct place:
The sentence True False
1- The number of students at first was 8 and then it declined.
2- The teachers at Babur’s school worked for free.
3- Babur’s parents were full of pride because of his success.
4- Creation was the secret of Babur’s success.
E) Find words in the passage with the SAME meaning
Word Synonym
tasks para.(6)
worries para.(7)
succeeding para.(9)
7- Read the following article about paddleboard racing then answer the questions that follow accordingly.
1. Paddleboarding is a mixture of two water sports, surfing and rowing. Paddleboarding uses a
surfboard and the paddleboarder ‘rows’ the board. However, there are two big differences. In
surfing, you have to stand but in paddleboarding you can kneel or lie on the board. In rowing
you use oars, but in paddleboarding you mustn’t use oars. You have to use your arms to move
along.
2. You can do the sport on rivers, but most of the big competitions are on the ocean. The main
competition for paddleboarders is the annual race from Molokai to Oahu in Hawaii. The
distance is 50 kilometres. On a good day, with the right kind of waves, you don’t always have to
use your arms because the water carries you some of the way but, on a bad day, you are using
your arms the whole way.
3. Competitors must be very strong and athletic. One of paddleboarding’s most famous
competitors is the Australian Jamie Mitchell. Not many people know about Mitchell, but he is
the eight-time winner of the Molokai to Oahu race. He also has the record time of four hours,
fifty- eight minutes and twenty-five seconds.
4. Because the sport isn’t well known, the prize money for winning paddleboarding is small
compared to other sports — Mitchell only received $3,000 for winning the race this year. But
Mitchell obviously loves the sport because he trains two or three times a day, six days a week,
for the four months before the race. At the same time, he has to earn money, so he does
anything including working in restaurants or building work.
5. So how does Mitchell stay interested in such a sport? He says, ‘I just love paddleboarding. It’s
not about winning. It’s about coming to Hawaii and spending time with my good friends in a
place that I love.’
A) Read the article and match the headings to the right paragraph. Note there is an extra heading
Heading Paragraph Number
A well-known athlete.
A little reward.
What is paddleboarding?
The most important race.
Advantages of paddleboarding
Socializing while practicing.
B) Read the following sentences and decide whether they are true, false or not given by putting tick (√) in
the correct place:
The sentence True False
1. Paddleboarding is a combination of two other sports.
2. Competitive paddleboard races are usually on rivers.
3. Jamie Mitchell completed the Molokai to Oahu race in the fastest time.
4. The writer says paddleboard racing is a famous sport.
5. Jamie is a full-time professional sportsperson.
8- Read the following advertisements and then answer the questions that follow accordingly
Tom's Trekking
Walk along mountain and forest paths with professional guides. The ideal pastime for nature lovers.
 Walk throughout the year, depending on the location.
 You must be in good physical condition.
Ages: 10-65; children must be accompanied by an adult.
Equipment & Cost:
1-day trek: a pair of trekking shoes, comfortable clothes, a waterproof jacket, a hat, sunglasses, a
backpack and a drinking flask; €25 for adults, €15 for participants aged 16 and under.
3-day trek: the same equipment as above, as well as a sleeping bag, a torch, a tent and camping
equipment; €60 for adults, €40 for those aged 16 and under.
River Rafting Adventures
Travel down fast-flowing rivers in inflatable boats with professional river guides.
 Usually October to May, though on some rivers you can enjoy rafting throughout the year.
 Start on easy rivers before attempting more challenging ones.
 You must be able to swim and be in good physical condition.
 Ages: 18+
 You will need a swimsuit, a windproof jacket, a T-shirt and towel. A neoprene suit, helmet, life
jacket and paddle will be provided. Contact us at river_rafting.com for price details

Rod's Rock Climbing Tours


Taught by professional climbing guides:
 Climbing is possible throughout the year.
 You should be in good physical condition and not suffer from a fear of heights.
 You should be between 18-60 Years old.
 You'll need comfortable clothes, a fleece jacket, a small backpack and a flask of water.
We provide climbing shoes, helmets, ropes and harnesses.
 Just €50 for a weekend; discounts for groups of 4 or more.

JUST MOUNTAIN
BIKING
Tour Mountain trails with bikes, accompanied by professional guides.
 Available throughout the year.
 If you know how to cycle and are in good physical condition, you can participate in this activity.
 For 10-65 year olds; children under 18 ride in their own group, accompanied by parents and a guide.
What to take with you: Warm clothes (during winter), a small backpack, a flask of water, a windproof
jacket, gloves and a hat. Don't forget! You must have your own mountain bike and helmet.
Cost: Call Mike on (020) 85101123, Mon-Fri 9-5, for more information
Eng. (102) Reflect Final Revision Activities Prepared by: Mr. Wael Bakry

A. Read the advertisements and Choose the correct answer a, b, or c.


1. For which activity you must not be afraid of high places.
a) 2 b) 4 c) 3
2. For which activity you must be a good swimmer?
a) 1 b) 2 c) 3
3. If you are under 18-year- old, which activities can't you do?
a) 1 and 2 b) 1 and 3 c) 2 and 3
4. Which activity sometimes includes camping?
a) 1 b) 3 c) 4
5. Two of the advertisements do not include contact information.
a) 1 and 2 b) 1 and 3 c) 2 and 3
B) Match the following words from the article with their definitions. Note that there is one extra word.
equipment - inflatable - attempting - suffer - discounts - participate
Word Definitio
n
1. an object that has to filled with air or gas.
2. to have an illness, health problem or injury.
3. to be involved by other people in an activity.
4. tools or things that are used for a particular activity.
5. when you try to do something.
C) Read the following sentences and decide whether they are true, false or not given by putting
tick (√) in the correct place:
The sentence True False NG
1- Physical fitness is important for trekking.
2- Elderly people can participate in short distance trekking.
3- River Rafting Adventures can be only during summer months.
4- You will need a thick jacket in Rod's Rock Climbing Tours.
5- If you are under 18, you should ride a bike with your leader.
9- Read the following article and then answer the questions that follow accordingly.
Prince Khalifa Bin Salman Park
1..........................................................?
Oh, it is as close as you can get to a park on a small desert island. It is impressive. There is a
lake, walkways, restaurants, shopping areas, seating areas and plenty of greenery. Around
six million dinars was spent creating this 80,000 square meter park- one of the biggest of its
kind in Bahrain- and it was seemingly worth it as it’s particularly popular on weekends for
families and friends.

2. ……………………………………………………?
Exactly, there’s plenty to keep the kids entertained, including a full-blown skate park, play
areas with swings and slides and other equipment, as well as a couple of kiddie rides and
ice cream. Lots of ice cream. For big kids, too, there is a games room complete with an air
hockey table to have a great time.
3. ……………………………………………………?
Definitely. You can go paddle boating, fishing, rent bicycles and ride around the dedicated
track, go jet skiing and, of course, walk around the special walkway. It was built to be an
outdoor haven of activity for leisure seekers and fitness enthusiasts alike.
4. …………………………………………………….?
You see the tower in the middle of the park? Well, there is a restaurant, Koffiatto, in there,
which offers a great 360-degree view of Bahrain’s skyline and the lake. There are also a few
food stalls serving local bites, plus tables, chairs and benches around the park so you can
have a seat and enjoy a quick bite.
5. ……………………………………………………….?
Well, there are a few rules worth noting down. For instance, do not take your dog for a walk
as animals are not allowed. In addition, cycling, fishing and skating are only allowed in the
designated areas. Moreover, please do not try to swim in the lake. Finally, do not mess up
the shrubs, plants and lawns by picking, tramping and playing ball sports. Let us keep it
green and clean so all people can enjoy it together.

A) Match the questions to the paragraphs. Note that there is an extra question.
Questions Paragraph Number
Are there ways to get some exercise in here?
What is there for my children to do?
Is there anything else I should know about the place?
What are the opening hours?
So is this a real park or just a “green space”?
All that activity is making me hungry. Where can I eat?
B) Read the following sentences and decide whether they are true, false or not given by putting
tick (√) in the correct place:
The sentence True False
1. Prince Khalifah Bin Salman park cost a big amount of money.
2. The park is suitable for all age groups.
3. The park is a good destination for exercise lovers.
4. Traditional food is not found at the park.
5. You can walk your pet at the park.
C) Match the following words from the article with their definitions. Note that there is one extra word.
walkway - kiddie - dedicated – stall - designated - bench
Word Definition
a young child.
a small shop with an open front from which goods are sold in a public place
chosen for particular purpose.
a passage or path, especially one that is covered or raised above the ground.
a long, usually hard seat for two or more people, often found in public
places.
D) What do the highlighted words in the passage refer to?
Word Reference
The pronoun 'its' para.(1) line (3)
The word 'there' para.(4) line (1)
The pronoun 'it' para.(5) line (4)
E) Find words in the passage with the SAME meaning
Word Synonym
mostly para.(1)
tools para.(2)
acceptable para.(5)
F) Find words in the passage with the OPPOSITE meaning
Word Antonym
different para.(3)
dislike para.(4)
arrange para.(5)
10- Read the following article and then answer the questions that follow accordingly.
The environmental Artists
Christo and Jeanne-Claude are a married couple, famous for their art installations all over the
world. Installation art is a special kind of art. Instead of making art on a canvas, like a painting,
the artists use the whole world as their canvas. They usually put the installation in a public
place, like a park. Christo and Jeanne-Claude are no exception, and they call themselves
“environmental artists”. Some of their most famous exhibits include: veiling (cover with fabric)
the Reichstag in Berlin in1995 and the Pont Neuf in Paris in 1985, setting up umbrellas in
public places in California and Japan at the same time in 1991, and wrapping almost 200
trees in a park in Basel, Switzerland in 1998.
One of their most successful installations was ‘The Gates’ in New York City’s Central Park. For
two weeks in February 2005, they hung more than 75,00 orange-colored fabric ‘gates’ over the
sidewalks of the park for people to walk through ‘The gates’ stretched for nearly 23 miles
and cost about $21 million dollars to create! Between 1991 and 1995, Christo and Jeanne-
Claude spent 39$ million to create their artwork. Part of the cost is for materials for the
creation and also fees and permits to use the land. Now, Christo and Jeanne-Claude have no
trouble creating their art, because the whole world wants to see what they are going to come
up with next!
A) Read the following sentences and decide whether they are true, false or not given by putting tick
(√) in the correct place:
The sentence True False
1. For installation art, the artists use only a canvas.
2. The artists usually put their installations in public places.
3. ‘The Gates’ were an installation in Switzerland.
4. It’s not expensive to make an installation.
5. People really love Cristo and Jeanne-Claude art.
B. Match the following underlined and highlighted words from the article with their definitions.
Note that there is one extra word.
exhibits - wrapping - successful - sidewalk - stretched - permits

Word Definition
Covering and protecting something using paper or plastic.
To become longer or wider.
Objects that are shown to the public such as painting flowers etc.
Achieving what you want to do.
A path with a hard surface by the side of a road that people walk on.
11- Read the following article and then answer the questions that follow accordingly.
1. The Remote Control

After long day at work or school, many of us come home, jump on the sofa and reach
for the remote control. For today's generation it is normal not to have to stand up to
change channels on the television or to turn on the music. This is, however, a relatively
recent invention. It was not until 1950 that the first commercial television remote
control was created. The Zenith Radio Corporation called it the 'Lazy Bone'. In those
days, however, the Lazy Bone was not wireless; it was attached to the television by a
long cable. This was seen as a danger, as people kept tripping and falling over it. Just
five years later, in 1955, the first wireless TV remote control was created by Eugene
Polley and this soon became very popular indeed. Opinions are divided as to whether
this invention is necessarily a good thing; many believe we have created a generation
of couch potatoes
- people who are too lazy to move from the sofa.

2. The Telephone

Alexander Graham Bell was born in Scotland in 1847. He was a very talented person; a
great inventor, an expert in speech and also very musical. His most famous invention
was, of course, the telephone in the 1870's. While experimenting with electrical signals,
he discovered he could hear a sound coming over the wire. On 10th March 1876 he
made the first telephone call to his assistant, Mr. Watson, in the next room. The very
first words ever spoken over the phone were 'Mr. Watson. Come here. I want to see
you.' The telephone has transformed the way the World communicates.

3. The Microwave

Dr. Percy L. Spencer invented the microwave oven by accident, in 1946. He was already
known as an electronics expert and worked with machines called magnetrons which
produced microwave radiation. One day, he stopped in front of one of these machines
when it was on and he noticed that a chocolate bar in his pocket had melted. Being an
experienced inventor, he decided to test his theory with popcorn and, sure enough,
when he placed the popcorn near the machine it became so hot it popped! From this
first experiment, it was clear that microwaves were able to heat food very quickly. This
is how the microwave oven was born.
A) Match the following questions with the correct paragraphs (1 – 3). Note that there is one
extra question.
Questions Paragraph Number
Caused accident at first, until it was changed?
Was created by mistake?
Was invented by someone who had many different abilities.
Used a hot liquid for power?
Needed two people to test it?
Is not always seen as a positive development?
B) What do the highlighted and underlined pronouns refer to?
Word Reference
The pronoun 'it' para.(1) line (5)
The word 'He' para.(2) line (1)
The word 'He' para.(3) line (1)
The pronoun 'it' para.(3) line (4)
The pronoun 'it' para.(3) line (6)
C) Answer the following questions:
1- Why was the Lazy Bone dangerous at the beginning?
…………………………………………………………………………………………………………………………………………………
2- Opinions are divided about the first remote control. Give reasons
…………………………………………………………………………………………………………………………………………………
3- Who was the first telephone call made to?
…………………………………………………………………………………………………………………………………………………
4- How did Dr. Percy know that microwaves were able to heat food very quickly?
…………………………………………………………………………………………………………………………………………………
D) Read the following sentences and decide whether they are true, false or not given by putting tick
(√) in the correct place:
The sentence True False
1- The first commercial television remote control was created before 1950.
2- Alexander Graham Bell is a multi-talented person.
3- Alexander Graham Bell was the first person to receive a phone call.
4- Dr. Percy invented the microwave oven after years of hard work.
5- The three inventions were made accidently.
12- Read the following article and then answer the questions that follow accordingly.
1. It seems we can’t get off the planet fast enough. Two thirds of Nasa’s annual budget
is devoted to manned space exploration, and that figure will grow with the USA’s
decision to send a man to Mars in 2037. We’ve seen all there is to see on Earth, right?
Wrong. The final frontier is here, beneath the surface of the sea.

2. Heading down into the ocean, human limits are quickly reached. At 200 metres, the
water is as black as a moonless night. Most nuclear submarines would break before
they reach one kilometre down. At three kilometres — still less than the average depth
of the ocean — there’s a good chance that you’ll discover a species completely new to
science. The deepest-diving whales go no further. At the very bottom, more than
eleven kilometres down, lie the Challenger Deeps. Twelve humans have walked on the
Moon. None has set foot in the deeps, and only two have seen them with their own
eyes.

3. Yet things live down there. Big things. Microphones throughout the sea listen for
enemy submarines, but no one has explained the undersea roar that occasionally
frightens listeners. The sound appears biological in origin, and its wavelength suggests
that it is produced by an animal bigger than a blue whale, the largest creature known
on the planet.

4. In the late I990s, a deep-water submersible was dropped in the Southern Ocean, and
passing 4,000 metres — well beyond the diving depth of any whale —it detected
something enormous passing beneath it. Surprised? Don’t be. The Ocean covers 70%
of the planet’s surface and We’ve investigated less than 5% of it. We know more about
the dark side of the Moon than about the bottom of the sea.

5. Satellites and unmanned space investigations allow us to look at our planet with a
global perspective and stare into the history of the universe. They’re absolutely vital for
doing ocean research too, but they can’t look under the sea, the only place where we
can search for clues to the origin of life itself. To do that, we need ships and
submersibles, manned and robotic. The cost of fuel-intensive marine expeditions is
rising, but the results would benefit all our lives. Understanding the oceans will give us
access to new sources of food, drugs and energy.
A) Read the following sentences and decide whether they are true, false or not given by putting tick
(√) in the correct place:
The sentence True False
1. There is no light 200 metres below the sea surface.
2. Submarines usually travel at depths of more than one kilometre.
3. No one ever been to the deepest part of the ocean.
4. Blue Wales are the largest living things on earth.
5. Exploration under the sea is cheaper than ever before.
B) Read the article again and match the following headings with the suitable paragraphs. Note that
there is an extra heading.
Headings Paragraph Number
A mysterious creature
A very beautiful place
Strange sound under water
Outer space travelling is on the rise
A place worth exploring
An extremely deep water
C) Match the following words from the article with their definitions. Note that there is one extra word.
devoted - moonless – submarine – suggest - beneath - bench
Word Definition
to mention an idea, possible plan, or action for other people to consider.
giving your time, effort to something you believe in.
the act of examining something carefully to discover the truth about it.
below
without light from the moon.
D) What do the highlighted words in the passage refer to?
Word Reference
The pronoun 'they' para.(2) line (2)
The word 'None' para.(2) line (7)
The pronoun 'its' para.(3) line (3)
The pronoun 'it' para.(4) line (2)
The pronoun 'They' para.(5) line (2)
13- Read the following article about “Installation Art” then answer the questions that follow
accordingly
1- Installation art is a kind of art that uses the area it is in to help people enjoy it. It isn’t
just about making paintings or sculpture; installation art includes exhibits with sound,
performances, video and even the Internet. The people who experience the art often
have to use different senses at once, not only sight, but also hearing, smell and touch.
You can find installation art in museums and in public places, like parks and shopping
centres where many people will see it. Most art installations are temporary and mostly
stay at the site only for a few weeks. The main point of installation art is for the people
not to just look at it, but to experience it and feel like they are part of the art, too.

2- These days there are many artists who make installation art. Carsten Holler
displayed his Test Site in the main hall of the Tate Modern in London from October
2006 to April 2007. Test Site was an installation of five large metal slides that people
could slide down for free.

3- Also in 2003 at the Tate Modern, artist Olafur Eliasson installed a glowing sun in the
room and a huge mirror on the ceiling. The room was also filled with a mist made of
sugar and water. The installation was called “The weather Project” and many visitors
enjoyed lying on the floor and looking up at their shadows on the ceiling.

4- Christo and Jeanne-Claude are French married couple who have made many famous
art installations all over the world. Some of their most famous exhibits include: covering
the Reichstag in Berlin and the Pont Neuf in Paris with fabric, placing umbrellas in
public places in California and Japan at the same time, and wrapping almost 200 trees
in a park in Basel, Switzerland. Their latest installation was “The Gates” in New York
City’s Central Park. For two weeks in February, 2005, more than 7500 orange-coloured
fabric ‘gates’ were hung over the pavements of the park for people to walk through.
The gates stretched for nearly 23 miles and cost about 21 million dollars to create.
A) Read the article and choose the correct answer a, b or c. (5 marks)
1. The purpose of installation art is to:
a) please people. b) educate people. c) hear people.
2. Installation art usually lasts for:
a) five days. b) few weeks. c) Few years.
3. Which artist(s) used light in their art display?
a) Carsten Holler. b) Olafur Eliasson. c) Christo and Jeanne-Claude.
4. The pronoun ‘their’ on paragraph 3 refers to:
a) sugar and water. b) visitors. c) shadows.
5. Christo and Jeanne-Claude come from:
a) France. b) The United State. c) Switzerland.
B) Match the following words from the article with their definitions. Note that there is one
extra definition.
Word temporary – artist – glowingDefinition
- couple - pavement
a person who produces art, especially paintings or drawings.
a promise or firm decision to do something.
existing or happening for only a short or limited time.
two people who are married or having a romantic relationship
to give out light and /or heat without smoke or flames.
the path at the side of the road that is for people to walk on.
C) Find words in the passage with the SAME meaning
Word Synonym
practice para.(1)
very big para.(3)
covering para.(4)
D) Find words in the passage with the OPPOSITE meaning
Word Antonym
exclude para.(1)
floor para.(2)
unknown para.(5)
14- Read the following texts and then answer the questions that follow accordingly.
A: Golf Ball Diver
Most people don't know what golf ball diving is, but it's actually big business and I am
making good money these days. Basically, I dive into lakes and ponds on golf courses to
collect golf balls that have fallen in so that I can sell them and make a profit. I travel to
different courses and, on average, I usually collect around 4,000 balls a day. Diving into
muddy waters, especially when there are snakes and crocodiles to think about, can be risky,
though! However, the biggest problem is created by people who break into golf courses at
night and steal the golf balls. I lose a lot of money because of them. All in all, I really enjoy
my job; I earn good money and it certainly beats working in an office.
B: Chef on a Submarine
Being a chef on a submarine is a very important and demanding job. The quality of the food
I serve affects the mood of everyone on board, especially when we are away at sea for
months at a time. I always have to be very imaginative with my recipes because the fresh
food runs out after a few weeks and then I have to use canned or frozen ingredients. I've
gotten used to it now, though, and I can make tasty meals out of almost anything. The
kitchen space is very limited and it gets really hot in there when I am cooking, so
occasionally I wish that I had a "normal" job, but most of the time I enjoy it. The hardest part
is being away from my family for such a long time. I think that's something you never get
used to. Anyway, at least my salary is good and I get long breaks in between jobs.
C: Ethical Hacker
I help companies all over the world protect themselves from online criminals who steal
private information from websites. I use similar methods to the criminals, only the hacking I
do is legal and even paid for. I attack companies’ websites and look for weaknesses that
criminals might use to gain access. This way companies can make their sites stronger and
keep illegal hackers away. Of course, it's a desk job, but I don't mind that, because I find it
exciting and I am always learning new technologies and techniques. At the moment, I am
working on a bank's website which has a really sophisticated security system, so it's a real
challenge for me to hack. I guess that's why I like what I do. In my opinion, life is boring
when it's too easy.
D: Sports Agent
Sometimes I can't believe how lucky I am; I get paid to do what I love. I have always been a
huge sports fan, but I’m also a real people person and love making new contacts and being
well-connected. That's why being a sports agent is the perfect job for me. It combines my
love of basketball with my personal skills. It can be tiring, as I often have to be available 24
hours a day for my clients and stay in touch with a wide network of contacts. However, I
love the fact that my days are never the same; some days I'm negotiating contracts with
lawyers and other days I'm organizing interviews on TV.
A) Read and match each of the questions below (1- 5) to people in the texts (A-D). Some
people may be chosen more than once.
Question Person
1. Whose job can be dangerous?
2. Who enjoys doing a challenging job?
3. Whose job combines interests with personal skills?
4. Whose job requires creativity and imagination?
5. Whose job constantly teaches new skills?
B) Match the following words from the article with their definitions. Note that there is one
extra definition.
Word limited - criminal - sophisticated - client - negotiate
Definition
very advanced
to arrange/ discuss
small in size
a person who breaks the law
somebody who pays to get the services of a professional company
to try very hard
C) Answer the following questions:
1- How is golf balls a good business for the writer?
…………………………………………………………………………………………………………………………………………………
2- Why does a chef have to be imaginative?
…………………………………………………………………………………………………………………………………………………
…………………………………………………………………………………………………………………………………………………
3- How can companies strengthen their sites and keep illegal hackers away?
…………………………………………………………………………………………………………………………………………………
…………………………………………………………………………………………………………………………………………………
4- Give one positive point and one negative point for a Sports Agent.
A) …………………………………………………………………………………………………………………………………………………

B) …………………………………………………………………………………………………………………………………………………
15- Read the following texts and then answer the questions that follow accordingly
A: Nick Noble – Barefoot skiing
When I tell people that my favorite hobby is barefooting, a lot of them think I enjoy walking
around without shoes or socks! But barefooting is far more exciting than that. Basically, its
water- skiing, but without the skis. Like water-skiing you hold onto a rope which is attached to a
motorboat, but because you don't wear skis (or anything else on your feet, the boat has to
travel at higher speeds (50-70 kmh), so that your body can stay up. How did I start? well, one
day I had planned to go water-skiing, but forgot my skis. Then, a stranger suggested I ski
without them. I hadn't even known that the sport existed until she told me, but I gave it a try -
and loved it immediately.

B: Karen Roe - Street luge


You might know luge from the Winter Olympics - the sport in which one or two people sit on a
sled and speed around an icy track. Well, I’ve never tried luge, but I do the next best thing -
Street luge! It's like luge, but you use a board with wheels, like a long skateboard, and race
along a street. You lie down on the board with your feet forward and you steer by moving your
weight from side to side. It’s quite dangerous, so always wear a helmet, and elbow and knee
pads; even with those I still end up with a few scratches! I also always have an extra strong pair
of shoes - because they’re my brakes!

C: Susan Ford - Parkour


I started parkour in my teens, and I have no plans to stop anytime soon! This extreme sport isn’t
very complicated - it just involves moving around an urban area as quickly and creatively as
possible! We use lots of different techniques including running, jumping, climbing and
balancing to move over or around rails, walls, benches and even pieces of public art! For me,
the great thing about parkour is that anyone can do it anywhere and at any time - and you
don't need to buy anything! Also, it's great exercise and an interesting way to discover your city.

D: David Taylor- Slacklining


This fascinating sport is extremely simple yet very challenging. The goal of slacklining is to walk
along a cord, or piece of flat woven fabric, that has been strung between two trees. Nothing
more, nothing less! As a general rule, the greater the distance, the more difficult it is to walk
from one end to the other. Trying slacklining for the first time was in 2017. Back then I didn’t
even think about taking it any further. However, a year later I decided to take it up as my hobby
and I bought a proper slackline set.
A) Read and match each of the questions below (1- 5) to the extreme sports in the texts (A-D). Some
sports may be chosen more than once.
Which extreme sport Extreme sport
1. doesn’t need any equipment?
2. do you practise without any footwear?
3. allows you to use your imagination?
4. is easy but challenging?
5. is considered to be road racing?
B) Read the following sentences and decide whether they are true, false or not given by putting
tick (√) in the correct place:
The sentence True False
1 Nick didn’t enjoy barefooting when he tried it for the first time.
2 Karen thinks that luging might be risky.
3 Karen often gets badly injured while street luging.
4 Susan loves parkour as it gives her good exercise.
5 David decided to take up slacklining as a hobby in 2017.
C) Match the following words from the article with their definitions. Note that there is one
extra definition.
Word Basically - skateboard - complicated - proper - balancing
Definition
involving a lot of different parts, in a way that is difficult to understand.
being in a position where you will stand without falling to either side.
in a way that involves or affects the fundamental character or condition.
difficult, in a way that tests your ability or determination.
real, satisfactory, suitable, or correct.
a flat, narrow board with two small wheels under each end, which a person stands on
and moves forward by pushing one foot on the ground.
D) What do the highlighted words in the passage refer to?
Word Reference
The pronoun 'them' para.(A) line (7)
The pronoun 'they' para.(B) line (7)
The pronoun 'it' para.(C) line (5)
The pronoun 'it' para.(D) line (5)
16- Read the following article and then answer the questions that follow accordingly.

1. Energy is very important in modern life. People use energy to run machines, heat or
cool their homes, cook, provide light, and transport people and products. Most energy
nowadays comes from fossil fuels - petroleum, coal, and natural gas. However, burning
fossil fuels causes pollution. Scientists are working to find other kinds of energy for the
future. What might these sources of energy be?

2. Energy from wind all over the world: People use the power of wind. It turns windmills
and moves sailboats. It is a clean source of energy, and there is lots of it, particularly in
countries such as the Netherlands and Denmark. Unfortunately, if the wind does not
blow, there is no wind energy.

3. Energy from water: When water moves from a high place to a lower place, it makes
energy. This energy creates electricity without pollution through the use of dams and
water turbines. Laos plans to build 55 dams and become the "battery" of southeast Asia.
Dams, however, cost a lot of money to build, so water energy is expensive.

4. Energy from the earth: There is heat in rocks under the earth. Scientists use this heat to
make geothermal energy. In Iceland, 87 percent of the population enjoys central heating
from this energy source - and it costs less than half the price of using oil for central
heating. Moreover, geothermal energy does not pollute. The problem is location - it's
only available in a few places in the world.

5. Energy from the sun: Solar panels on the roofs of houses can turn energy from the
sun into electricity. These panels can create enough energy to heat or cool an entire
house. In fact, some scientists say that if we build solar panels in just 1 percent of the
Sahara Desert, in countries such as Algeria and Libya, there will be enough electricity for
the entire world. However, solar energy is expensive to export.

6. Energy from living organisms: Living organisms, such as plants and algae, can produce
energy that is called biofuel. In 2003, Brazil started manufacturing “flex-fuel" engines -
they can run on gasoline or biofuels. Biofuels are renewable and they don’t cause
pollution. For example, sugarcane (used to make ethanol) can be produced every year.
But some people say we should use farmland to produce food, not biofuels.
A) Read the article and answer the following questions.
1. Mention two different uses of energy in today’s life.
A) …………………………………………………………………………………………………………………………………………………
B) …………………………………………………………………………………………………………………………………………………
2. What’s the main disadvantage of using wind energy?
…………………………………………………………………………………………………………………………………………………
3. How does water produce energy?
…………………………………………………………………………………………………………………………………………………
4. What is geothermal energy used for in Iceland?
…………………………………………………………………………………………………………………………………………………
5. What does “they” in paragraph 6 refer to?
…………………………………………………………………………………………………………………………………………………
B. Match the following words from the texts with their definitions. Note that there is one extra
definition.
Word pollution - source - dam - Definition
solar-panel - algae
A device that changes energy from the sun into electricity.
Damage caused to the environment by harmful substances.
The place something comes from or starts at
A strong wall built across the river to stop the flow and collect water
Small plants that grow in or near water.
Power from something
C) Read the following sentences and decide whether they are true, false or not given by putting tick
(√) in the correct place:
The sentence True False NG
1- Scientists are working to modify fossil fuel to be clean energy.
2- Netherlands and Denmark have so much water power.
3- Geothermal energy is a cheap and clean source of energy.
4- The cost of solar panels is so expensive.
5- “Flex-fuel" engines run on both solar energy or biofuels
17- Read the following extracts and then answer the questions that follow accordingly.
A: Tony
My sleep problem is really quite serious, but a lot of people think that it’s normal and even
funny. I snore at night when I’m asleep – and my wife says that it’s pretty loud! Now, I know
snoring isn’t unusual and there is a lot of advice on how to stop someone snoring, but my
snoring is different! It’s linked to something called sleep apnoea. When you snore it means that
you can’t keep your throat open when you’re asleep. In my case, I actually stop breathing for
ten seconds
or more. It worries my wife but I always wake up afterwards!
B: Lizzie
My problem is very unusual and most people have never heard of it. It’s called narcolepsy and
it has changed my whole life. Narcolepsy is when people fall asleep anywhere at any time and
they can’t stop it. Since the problem started I’ve had to stop working because you can’t do a
good job if you fall asleep in the middle of a conversation! My friends have been very
supportive but I don't like going out now because I get embarrassed about it and also I’ve had
to stop driving as
it’s much too dangerous. I go to a special sleep clinic now and they’re helping me a lot.
A. Read the extracts and decide which person each statement best describes. (Write: Tony, Lizzie or
C: Robert
Robert). you may use a name more than once.
I’ve had this sleep problem since I was a child and I know a lot of people who have the same
problem. It’s called insomnia andStatement
it means I sleep very badly. Sometimes I can’tPerson
get to sleep
hasn’t
for hoursdone
andwhat the doctor
sometimes I gosuggested
to sleep OK, but I wake up in the middle of the night and then I
has a rare problem
can’t get back to sleep again. It’s horrible because the next day I can’t concentrate on my
has had
work. Thethe problem
doctor gavefor
mea sleeping
long timetablets, but I don’t like taking them because they make me
is getting
feel bad thebetter
next day.
thinks some people laugh at his/her problem
A. Read the extracts again and decide whether the statements are True (T), False (F), or
No information (NI):
The sentence True False NI
1. Tony makes a lot of noise when he snores.
2. Tony's wife finds it difficult to wake him up.
3. Lizzie doesn't mind having narcolepsy.
4. Lizzie sometimes falls asleep when she's talking.
5. Robert's problem has negative effects on his job.
18- Read the following magazine article about a young mother whose house was burgled
and then answer the questions that follow accordingly.
Lisa Tyler was weary after a long, hard day at the pottery factory where she works. But as
she approached her home in the English city of Stoke-on-Trent, her heart lightened.;
soon she would be having a nice cup of tea, putting her feet up and watching Friends,
her favourite TV series.

As Lisa walked up her garden path, she noticed a light flashing on and off in an upstairs
bedroom. A shiver went down her back. What if it was a burglar? Quietly, she crept round
to the back of the house to see if there was any sign of a break-in. Sure enough, a
window was open and someone’s coat was hanging on the gatepost!

Well, 26-year-old Lisa didn’t fancy coming face to face with a burglar, so she ran to a
neighbor’s house and rang the police. But as she sat waiting for the police to arrive, Lisa’s
curiosity got the better of her and she decided to go back and see what was going on.
That’s when she saw a leg coming out of the downstairs front window. It was a man
climbing out. Lisa gasped in shock. The burglar was carrying her portable television!

At this point, Lisa saw red. She didn’t have many possessions and she’d saved long and
hard to buy that set. Besides, nobody was going to stop her watching Friends.
Without even stopping to think, she tore across the garden and started shouting at the
burglar. “Give me my TV - drop it now! “she screamed”.

Ignoring her, the man fled across the garden. So Lisa threw herself at him and
successfully rugby- tackled him to the ground. The burglar struggled to escape, but Lisa
hung on like the best kind of guard dog despite being punched and kicked. As she
looked up, she realised that she recognised the burglar’s face. She was so surprised that
she lost her grip and burglar got away, leaving the TV behind him.

By the time the police and her father arrived, Lisa was in tears. 'I can’t believe you were
so foolish, Lisa,' scolded her father. 'You could have been killed. '

“I know, but at least he didn’t get my TV,” she replied.

Lisa later remembered the name of the burglar, who had been in the same year as her at
school. He was later caught and jailed for 15 months after admitting burglary and
assault.
A) Read the article and choose the correct answer a, b or c. (10 marks)
1. How was Lisa feeling as she walked home from work?
A tired
B anxious
C depressed
2. What first led Lisa to think there was a burglar in her house?
A something had been broken.
B something had been left outside.
C something was moving inside.
3. Why didn’t Lisa wait in her neighbor's until the police arrived?
A she was worried about losing her television.
B she wanted to know what was happening.
C she noticed something from her neighbor's window.
4. What happened when Lisa shouted at the burglar?
A he fell over as he ran towards her.
B he pretended not to have heard her.
C he dropped the TV and attacked her.
5. What did Lisa’s father do when he arrived?
A he spoke to her angrily.
B he comforted her.
C he praised her.
B. Match the following words from the texts with their definitions. Note that there is one extra
definition.
Word break-in - gasped - possessions - burglar - assault
Definition
a person who illegally enters buildings and steals things.
to enter a building by force.
to take a quick deep breath with your mouth open especially because of
shock
or pain.
something you own
a violent physical attack
to shout in a loud voice
19- Read the following text and then answer the questions that follow accordingly.
1. Feeling a bit stressed about something? Well, you’re not alone. In the modern World, it’s
almost impossible to avoid stress. For adults, there are the pressures of work, while teenagers
often suffer from exam stress. And now we know that stress can have terrible effects on our
health. So, what can we do to reduce it?
2. One of the best tools to reduce stress is music. Research has shown that listening to calm
classical music reduces stress hormone levels in the blood, while a study from the Royal
Northern College of Music found that singing can reduce stress – even if you have a terrible
voice! Generally, music produces positive brain neurotransmitters, which makes you feel more
relaxed.
3. Another way to deal with stress is to take cold showers. According to the Dutch extreme
athlete Wim Hof, who made the method popular, having a cold shower forces the body to feel
extremely stressed, so any other stress that you feel later in the day doesn’t feel as bad! Also,
cold showers make your heart and lungs work better.
4. When it comes to beating stress, nothing beats exercise. When we exercise, our bodies
release hormones called endorphins, which make us feel happy. Doing exercise also makes us
feel that we’ve achieved something, which psychologists say is very important for overall
happiness. For best results, you should exercise for around 150 minutes a week – broken up in
two or three separate sessions.
5. Another solution is to change the way we think about stress. Many experts believe that there
is nothing wrong with having a little stress – and that it can actually be good for us! When we’re
stressed, our heart beats faster and more blood goes to the brain and muscles. This can actually
help us deal with our problems. So, as long as your stress isn’t from a long- term problem,
maybe you should ‘use’ your stress instead of worrying about it.
A. Read the article and match the headings to the right paragraph. Note there is an extra heading.
Headings Paragraph Number
Physical training is really useful
Stress is not the monster you think it is
Relax with tones
No one can escape from stress
An extreme method
Stress in the past
A) Read the following sentences and decide whether they are true, false or not given by putting tick
(√) in the correct place:
The sentence True False
1. Only good singing reduces stress.
2. Cold showers can harm your heart and lungs.
3. Endorphins are hormones that improve our mood.
4. Experts advise people to exercise two or three times a week.
5. Experts believe that stress may have some good effects.
20- Read the following text and then answer the questions that follow accordingly.
1- Last week when I received acceptances from my top two choices for college, State and
Greenwell, I knew I had a difficult decision to make. Although I had talked to friends and
relatives who had attended both schools and I had visited both campuses many times, I
couldn't make up my mind. It was only after I compared the location, cost, and quality of
education of the two schools that I could finally come to my decision to attend Greenwell.

2- The first thing I considered was the location. First of all, both universities are located in
Pennsylvania, where I am from. But that is where the similarities end. State's setting in a safe
suburb is definitely more appealing than Greenwell's location in a dangerous city
neighborhood. I also like State's older campus with its beautiful buildings and gardens more
than Greenwell's new campus, which looks like an office complex.

3- In addition to location, I had to pay a lot of attention to the financial component. The
tuition is the same at both schools -$20,000 per year. However, Greenwell offered me a
$3,000 scholarship, but State couldn't give me any money. Also, if I go to Greenwell, I can
live at home and save money on room and board. Finally, since Greenwell is much closer to
home, I won't have to spend as much on transportation home during vacation breaks.

4- The quality of education at the two schools had the most influence on my decision. In
many ways, State and Greenwell have similar standards of education. Both have large
libraries and excellent academic reputations. Also, State has a first-class engineering
department, and so does Greenwell. So I had to look at other things. What it came down to
was the difference in the class size between the two universities. State has large classes and
an impersonal feeling. On the other hand, Greenwell has small classes, and students get a lot
of personal attention.

5- Taking everything into consideration, I think I made the right decision. Since small classes
and personal attention from my professors are very important to me, I will probably be
happier at Greenwell
A) Read the text and choose the correct answer a, b or c (10 marks)
1 Which university offered the writer a seat to study?
A. State University B. Greenwell University C. Both
2 Which university is located in a secure surrounding?
A. State University B. Greenwell University C. Both
3 Which university doesn’t offer financial help for students?
A. State University B. Greenwell University C. Both
4 The school that has a high reputation is/are…
A. State University B. Greenwell University C. Both
5 The school that gives individual attention to students is/are…
A. State University B. Greenwell University C. Both
B) Match the following words from the texts with their definitions. Note that there is one extra
definition.
vacation - tuition - suburb - campus - academic

Word Definition
Buildings belonging to a university.
Money paid for college, university, or school lessons.
Relating to education like schools, universities etc.
A period of holiday.
A living area away from the city.
The opinion that people in general have about someone or something
C) What do the highlighted words in the passage refer to?
Word Reference
The pronoun 'its' para.(2) line (4)
The word 'Both' para.(4) line (2)
D) Find words in the passage with the SAME meaning
Word Synonym
site para.(1)
element para.(3)
choice para.(4)
E) Find words in the passage with the OPPOSITE meaning
Word Antonym
rejections para.(1)
pleasing para.(2)
personal para.(4)
21- Read the passage and answer the following questions:
Three of the most famous criminals in history
Harold Shipman
Harold Shipman, also known as “Dr. Death,” is believed to have killed at least 218 patients,
although the total is quite likely closer to 250. This doctor practiced in London and between
1972 and 1998 worked in two difference offices, killing all the while. He was not caught until
a red flag was raised by several people, including an undertaker who was surprised by the
absolute number of burial certificates Shipman was a part of, along with the fact that most
of the cases were elderly women found to have died in bed not at night but rather during
the day. Police mishandled the investigation, and Shipman kept killing until he got greedy
and tried to fabricate a will for a victim that named him beneficiary, which led the victim’s
daughter to become suspicious. He was finally convicted in 2000 and committed suicide in
prison.
Pedro Lopez
One of the world’s most creative serial killers might still be out there. Pedro Lopez is linked
to more than 300 murders in his native Colombia and in Ecuador and Peru. At least one-
third of those murders were tribal women. After Lopez’s arrest in 1980, police found the
graves of more than 50 of his preteen victims. He was later convicted of murdering 110 girls
in Ecuador and confessed to 240 more murders in Colombia and Peru. The “Monster of the
Andes” did not even spend 20 years in prison, as he was released in 1998 for good behavior.
Ted Bundy
Ted Bundy loved the attention his murders garnered him, and many in the United States
were more than happy to give him that attention. The western U.S. was his hunting ground,
with an unknown number of murders piling up—mostly college-age women—from
Washington and Oregon all the way to Utah and Colorado. Bundy was once arrested in
Colorado and convicted of kidnapping, but he escaped custody, moving to Florida where he
killed multiple times more. Bundy’s final arrest and its aftermath captured the attention of
the nation, as the accused murderer acted as his own lawyer during what is believed to have
been the first televised murder trial, welcomed interviews. He was eventually executed in an
electric chair in 1989.
John Wayne Gacy
A construction worker known by his suburban neighbors as outgoing, John Wayne Gacy was
involved in politics and even acted as a clown for birthday parties. He was no clown. Gacy
came under suspicion in 1978 when a 15-year-old boy, last seen with him, went missing.
That was not the only time families of missing boys had pointed fingers at Gacy, but it was
the first time authorities took them seriously. Soon after, a search warrant granted police
access to the Gacy home, with the smell of nearly 30 bodies buried in a four-foot crawl
space under his home. He was convicted of 33 counts of murder, with additional counts of
torture, and was executed in 1994.
A) Match the description to the suitable criminal:

Action Criminal
Multinational killer finally was set free after jailing in unisex crimes.
People doubted him because of the disappearance of a teenager.
A criminal physician caught due to his victim's relative doubt.
Had special consideration and sentenced after killing and hijacking.
B) Answer the following questions:
1. Why was Harold Shipman called Dr. Death?
…………………………………………………………………………………………………………………………………………………
2. What punishment did Ted Bundy have?
…………………………………………………………………………………………………………………………………………………
3. That was not the only time families of missing boys had pointed fingers at Gacy.
'Pointed fingers at' here means ……..
A) waved B) accused C) warned
4- Find a word in the passage that means 'a child from 11 to 12 years' ……………………..
C) Match the words to the correct definition:

a)
b)
undertaker – mishandle – convict – custody - execute

Word Definition
to deal with something without the necessary care or skill.
the state of being kept in prison, while waiting to go to court for trial.
to kill someone as a legal punishment
a person whose job is to prepare dead bodies that are going to be buried.
to decide officially in a law court that someone is guilty of a crime.
D) Read the following sentences and decide whether they are true, false or not
given by putting tick (√) in the correct place:
The sentence True False Not Given
1- Most of Harold Shipman victims were elderly people. 
2- Pedro Lopez killed most of his victims by his neighbour's help. 
3- Ted Bundy killed many times in Florida. 
4- John Wayne Gacy was convicted with two types of crimes. 
E) Which one of the four criminals do you think is the cruelest? Why?
………………………………………………………………………………………………………………………………………………
………………………………………………………………………………………………………………………………………………
22- Read the passage and answer the following questions:

Galileo Galilei: Discoverer of the Cosmos

1- Around Dec. 1, 1609, Italian mathematician Galileo Galilei pointed a telescope at the
moon and created modern astronomy. His following observations turned up four satellites
— massive moons — orbiting Jupiter, and showed that the Milky Way is foggy light shines
from many dim stars. Galileo also found sunspots upon the surface of our star and
discovered the phases of Venus, which confirmed that the planet circles the sun inside
Earth’s own orbit.

2- The 45-year-old Galileo did not invent the telescope, and he was not the first to point
one at the sky. However, he is undeniably one of the greatest scientists of all time: his
conclusions changed history. Galileo knew he’d found proof for the theories of Polish
astronomer Nicolaus Copernicus (1473-1543), who had launched the Scientific Revolution
with his sun-centered solar system model.

3- Galileo’s work was not all staring at the sky, either: His studies of falling bodies showed
that objects dropped at the same time will hit the ground at the same time, excepting air
resistance
— gravity doesn’t depend on their size. And his law of inertia allowed for Earth itself to rotate.

4- However, all this heavenly motion contradicted Roman Catholic doctrine, which was
based on Aristotle’s incorrect views of the cosmos. The church declared the sun-centered
model heretical, and an inquisition in 1616 ordered Galileo to stop promoting these
views.

5- The real blow from religious officials came in 1633, after Galileo published a comparison
of the Copernican (sun-centered) and Ptolemaic (Earth-centered) systems that made the
latter’s believers look foolish. They placed him under house arrest until his death in 1642,
the same year Isaac Newton was born.

6- The English mathematician would build on Galileo’s law of inertia as he compiled a set of
laws so complete that engineers still use them centuries later to navigate spacecraft across
the solar system — including NASA’s Galileo mission to Jupiter.

A) Match the heading to the suitable paragraph:

Heading Paragraph number


The church banned his views
His theories are still used
He wasn’t the real inventor of the telescope
His telescope discovered many objects
A contrast that ended everything
His work was more than that
B) Rewrite the following sentences correctly:
1. Galileo also found sunspots upon the surface of our star and discovered Venus.
…………………………………………………………………………………………………………………………………………………
2. Galileo did not invent the telescope, but he was the first to point one at the sky.
…………………………………………………………………………………………………………………………………………………
3. The church promoted the ideas of Galileo and honored him.
…………………………………………………………………………………………………………………………………………………
4- The English mathematician would refuse Galileo’s law of inertia.
…………………………………………………………………………………………………………………………………………………
C) Match the words to the correct definition:

c)
dim – undeniably – rotate – contradict - compile

Word Definition
to turn or cause something to turn in a circle, especially around a fixed point.
to say the opposite of what someone else has said.
not giving or having much light.
to collect information from a variety of places and arrange it in a book.
cannot be doubted.
D) What do the highlighted words in the passage refer to?

Word Reference
The word 'one' para.(2) line (1)
The pronoun 'his' para.(2) line (4)
The pronoun 'their' para.(3) line (3)
The pronoun 'They' para.(5) line (3)
The pronoun 'them' para.(6) line (2)
E) Find words in the passage with the same meaning:

Word Synonym
huge para.(1) massive
evidence para.(2) proof
principle or policy para.(4) doctrine
supporting or encouraging para.(4) promote
a group of para.(6) a set of
23- Read the passage and answer the following questions:

What is creativity and innovation?

1- Creativity is when people use their imagination to create new ideas, solve problems and
think of possibilities that no one else has thought of before. The scope of creativity is limited
only by one’s ability to think outside the norm. The nature of creativity means that an idea's
creation is unique and original to the creative thinker. Creativity is not a genetic feature, but
something that a person develops as they continue to learn and grow and use their
imagination for various forms of expression. Creativity has no inherent value unless a person
manifests it into reality.

2- Innovation is the process of taking creative ideas and exploiting their benefit for
commercial and financial success. An innovation is the creation of an innovator. Since
innovation is the application of creative ideas, it is naturally connected to creativity and they
both work in cycle with one another. An innovation can be a physical object and a concept
to improve or create a new process. People innovate to solve a problem or make society
functionally more convenient.

3- Creativity and innovation are important because they allow the world to grow, develop
and change over time. Innovation cannot happen without creativity, and creativity is light if
it is not applied. They are both integral to the progression and advancement of human
civilization as people think of new ways to express their ideas and develop those ideas into
innovations that make society safer, healthier and more accessible. Creativity and innovation
have benefited society via the invention of medicine, music, communication, transportation
and art.

4- An invention is an idea developed into something that society can use. Inventions are
novel and can take the form of a process, method or device. The meaning of invention and
innovation sometimes blend together, but they are clearly their own. An invention is just the
creation of an idea into physical form. The creation of an invention does not guarantee its
usefulness or functionality. An innovation is an invention created in business and sold for
profit and widespread consumption. You cannot have an innovation without the invention
itself.

5- Internet search engines are one of the most well-known examples of current innovation.
The global success of the cellphone changed the way people communicate. Cellphones
continue to evolve from modern era innovations and ideas.
A) Answer the following questions:
1. What does the nature of creativity mean?
……………………………………………………………………………………………………………………………………………………
2. What is the relation between creation and innovation?
……………………………………………………………………………………………………………………………………………………
3. Why are creativity and innovation important?
……………………………………………………………………………………………………………………………………………………
4- Why cannot you have an innovation without the invention itself?
……………………………………………………………………………………………………………………………………………………
B) Match the words to the correct definition:

d) manifest– exploit – integral – via - blend


e)

Word Definition
to use something in a way that helps you.
going through or stopping at a place on the way to another place.
to mix or combine things together
to show something clearly, through signs or actions.
necessary and important as a part of a whole.
C) Read the following sentences and decide whether they are true, false or not
given by putting tick (√) in the correct place:
The sentence True False Not Given

1- Creativity is limited only by one's ability to think inside the norm.


2- Scientists are the most creative people in their society.
3- Invention and innovation sometimes mixed together.
4- Internet search engines are one of the most well-known
examples of modern innovation.
D) What do the highlighted words in the passage refer to?

Word Reference
The pronoun 'it' para.(1) line (6)
The pronoun 'their' para.(2) line (1)
The pronoun 'they' para.(3) line (3)
The pronoun 'its' para.(4) line (4)
24- Read the passage and answer the following questions:

Sleep
1- Sleep is an important part of our lives. It improves physical and mental health. Sleep
happens in stages, including REM (rapid eye movement) sleep and non-REM (non-rapid eye
movement) sleep. Many factors affect your sleep quality, including food and room
temperature.

2- You may think nothing is happening when you sleep. However, parts of your brain are
quite active during sleep. Moreover, enough sleep or lack of it affects your physical and
mental health. When you sleep, your body has a chance to rest and restore energy. A good
night’s sleep can help you cope with stress, solve problems or recover from illness. Not
getting enough sleep can lead to many health concerns, affecting how you think and feel.

3- We need it for growth, in children and young adults; deep sleep (sleep that is harder to
wake from) supports growth. The body releases growth hormone during this type of sleep.
The body also increases production of proteins, which we need for cell growth and to repair
damage. Sleep is important for Well-being as people who do not get enough sleep are at
higher risk for developing various health conditions including obesity, diabetes and heart
problems.

4- We need sleep for survival, as researchers do not fully understand why sleep is so
essential, but studies in animals have shown that getting deprived of REM sleep can shorten
lifespans. Lack of sleep may harm the immune system, which protects us from infections.

5- Good sleep habits, also called good sleep hygiene, are practices to help you get enough
quality sleep. We need to have a sleep schedule, so try to go to sleep and wake up around
the same time every day, even on weekends and vacations. Another important thing is
clearing your mind before bed by making a to-do list early in the evening, so you will not
stay awake in bed and worry about the next day.

6- Creating a good sleep environment is a good thing, as you have to make sure your bed
and pillows are comfortable. Turn down the lights and avoid loud sounds. Keep the room at
a comfortable temperature. Also, exercise every day to stay active but try to avoid exercising
during the few hours right before bed.

7- Finally, you have to relax before bed, take a warm bath, read or do another relaxing
activity. Last but not least, see your healthcare provider if you have been having trouble
sleeping or feel extra drowsy during the day, talk to your provider.
A) Match the heading to the suitable paragraph:

Heading Paragraph number


A wrong thought
Suitable atmosphere is needed
Important for building up body
Things you have to do before sleep
Two types of sleep
The effects of poor sleep
You have to arrange your sleep
B) Answer the following questions:
1. What is the difference between REM and NREM sleep?
REM sleep: ………………………………………. NREM sleep: ……………………………………………………….
2. Mention TWO benefits of good night sleep.
a) ………………………………………………………………. b) ……………………………………………………………….
3. What is the medical explanation of deep sleep?
……………………………………………………………………………………………………………………………………….
4- What are the two good sleep habits the writer mentioned in the passage?
a) ………………………………………………………………. b) ……………………………………………………………….
C) Match the words to the correct definition:

f) factor– restore – various – hygiene - drowsy


g)

Word Definition
the degree to which people keep themselves clean to prevent disease.
a fact or situation that influences the result of something.
being in a state between sleeping and being awake.
many different.
to return something or someone to an earlier good condition or position.
D) Read the following sentences and decide whether they are true, false or not given by putting
tick (√) in the correct place:

The sentence True False NG


1- Children, young adults and elderly people need sleep to grow.
2- Good sleep hygiene is the same as good sleep habits.
3- The atmosphere of the room is important for good sleep.
4- Relaxing activity is not important before sleep.
25- Read the passage and answer the following questions:
Scary stories about night shifts
1- “I worked as an evening manager at a major Hilton property. I got a complaint from a
bunch of guests about the noise coming from one of the rooms. Turns out, a drunken man
giving a severe beating to his wife and had the door lock up, so we could not get in. I called
the cops, and they had to get into the room using the balcony from the room next room
over. I am still shocked by what I saw when they finally arrested him and got the wife out of
the room. She was covered in blood. It was horrifying."

2- “I worked overnight in a very old but still functional hospital. The building was in bad
condition but still had patients and staff. Therefore, I was clearing out this old closet when
the door behind me, suddenly, was shut. It was wedged open with a fire extinguisher. I did
not hear anything or see anyone. I switched my phone torch and left the room. So, I went
down to the security office and reported the incident. They played the CCTV footage, saw a
person, dressed all in black with bare feet, just walks up silently to the cupboard, and grabs
the fire extinguisher. The security fellows flew into action but the guy was never caught.”

3- “I worked in a restaurant on the main strip of bars in my college town. The night shift was
a breeze generally, but when the pandemic hit, we were the only place still open and the
street looked like a ghost town. One night, I ended up having to wait completely alone for
the last hour because my “stalker” was the last order of the night. He was consistently late
to pick up his food but usually, I at least had my boss with me. He used to record me while I
was working and send it to me on social media that I had never given him and he knew my
birthday before I ever met him. That was the longest hour of my life.”

4- When I was 18, I got a gig. I was working as a security guard and while doing my nightly
patrols at around 2 am, I came across a sweet old lady, sitting in her wheelchair, outside of
her room. She waved me over and asked me if I can tell the children to stop running in the
halls because she was trying to sleep. I resumed my patrols looking and listening for these
kids and found nothing. I returned back to the main nurse’s desk and told them what had
happened. They stared at me and told me that people see and hear weird things at night. It
turns out that the place was an old orphanage in the 30s that had burned down and killed a
few kids.”
A) Match the description to the suitable criminal: (There is one extra sentence)

Sentence Story number


The person in black was seen running through the cam
People fought him after long chase
He met all ghosts that night
The fight was violent and horrifying
Someone was following him at night
B) Read the passage again and choose the correct answer:

1- The cops had to get into the room using the balcony because the drunken man ...
a) was beating his wife b) closed the door firmly c) had a weapon
2- The person, dressed all in black, took …………….. .

a) fire extinguisher b) the CCTV c) the cupboard


3- It was strange that the stalker send him his picture in social media because …..
a) his boss wasn’t here b) he never met him c) he had never given him
4- The old lady, sitting in her wheelchair, outside of her room was ………….
a) a ghost b) nurse c) orphan
C) Match the words to the correct definition:

h) severe – extinguisher – consistently – weird - orphanage


i)

Word Definition
very strange and unusual, unexpected, or not natural.
causing very great pain.
a home for children whose parents are dead or unable to care for them.
in a way that does not change.
a device that discharges a jet of water, or other material to put out a fire.
D) Find words in the passage with the same meaning

Word/phrase Synonym
a group of para.(1)
videotape para.(2)
cupboard para.(2)
job para.(4)
continue para.(4)
26- Read the passage and answer the following questions:

Scary stories
1- Humans Can Lick Too
A girl is alone at home one night, for the first time. She has a dog to keep her company. The
girl listens on the news about a serial killer on the loose. She goes to sleep after locking all
the doors and windows but for one, which doesn’t close. Suddenly she is awakened by a
dripping sound. Scared thoroughly, she tries to go back to sleep and puts her hand down
the bed for a reassuring lick from her dog. As the dripping sound continues, she goes to
check on the taps in the kitchen, bathroom and everywhere else but finds nothing.
She returns to her bedroom and puts the hand down again for the dog to lick. The dripping
sound continues and the girl is too frightened to fall asleep. But as her hand gets licked
from under the bed, she slowly falls asleep. In the morning, when she opens the closet, she
finds her dog killed and hanged upside down. The dripping sound was from the dog’s
blood. On the closet doors, she sees a message – “Humans can lick, too”.

2- Aren’t You Glad, You Didn’t Turn On The Lights?


Two best friends Meg and Samantha, live in the same room of the college dormitory.
Samantha goes out for a party while Meg stays back to study for the upcoming mid-term
exams. When Samantha returns from the party, lights are all turned off. She does not switch
on the lights as her friend is asleep.
Early in the morning, Sam tries to wake Meg up to discuss the exams. As Meg does not
respond, Sam hesitantly uncovers her face to wake her up. She gets horrified to find Meg’s
face turn black and blue, and falls on the floor in a state of shock. She observes that Meg’s
study table was messed up and finds a note on the wall, “Aren’t you glad you didn’t turn on
the lights?” The murderer must have very much been in the room when Sam came the
previous night.

3- The Vanishing Hitch-Hiker


A man is driving home late in the night when he spots a girl asking for a hitchhike. The
pretty girl is dressed in a beautiful white dress. The man offers her a ride and they strike up
an interesting conversation. He drops the girl at her home. Next day, while driving for work
he notices that the girl by accident has forgotten her sweater in his car. He drives towards
her home to hand over the sweater. An old lady opens the door when he rings the bell. He
narrates the incident, which occurred last night and gives the sweater to the lady.
The lady refuses to accept it, saying he is mistaken. The man is surprised and questions the
lady again. He is astonished when the lady says her daughter died in a car accident a couple
of years ago.
A) Match the situation to the correct story :(A story can be written more than one time)

Situation Story name


She was notified about something she didn’t want to do
People did not believe what he/she said.
She knew about the thief from the news.
She disappeared when he went to give her clothes back.
The criminal hanged the victim.
B) Match the words to the correct definition:

j)
k)
dripping – dormitory – hesitantly – hitchhike - narrate

Word Definition
a large room containing many beds, for example in a boarding school.
to travel by getting free rides in someone else's vehicle.
to tell a story, or to describe events as they happen.
in a way that is not immediate or quick because you are nervous.
the sound of water drops.
C) Read the following sentences and decide whether they are true, false or not given by
putting tick (√) in the correct place:

The sentence True False Not Given


1- The dripping sound was from the bleeding dead dog.
2- At the end, the dog licked the girl's hand and she slept.
3- The murderer spent so long time in Meg's room.
4- The boys in story three all were killed by a dangerous criminal.
D) Find words in the passage with the same meaning

Word/phrase Synonym
extremely para.(1)
disorganized para.(2)
surprised para.(3)
E) What do the highlighted words in the passage refer to?

Word Reference
The word 'one' para.(1) line (3)
The pronoun 'her' para.(1) line (7)
The pronoun 'She' para.(2) line (6)
The pronoun 'they' para.(3) line (2)
27- Read the passage and answer the following questions:
New Job? How to Survive First-Week Challenges
1- It is natural to be nervous during your first week at a new job, and challenges will almost
certainly arise. One of the most difficult aspects of starting a new job is having to quickly
catch up to the rest of your team, especially if you’re replacing someone. While good
managers are understanding and expect there to be a learning curve, the business cannot
pause for too long. Therefore, new hires often find themselves bombarded with tons of
information and details almost immediately.

2- While some new employees do face an overload of work, others face the opposite
problem: not enough of it. If your manager or fellow team members have a particularly
packed schedule the week you start, it is likely you will end up doing some basic tasks that
do not have a lot to do with your overall job.

3- When you are excited about your new job, it is natural to want to start contributing ideas
right away. Participating from the very start shows you are interested in collaborating with
your colleagues and helping the company. But there’s a fine line between confidence in your
ideas and arrogance. In fact, employees who have been at the company a while tend to not
like the beginner who thinks they already know everything even though they just started.

4- It is likely many of your new co-workers will be friendly and introduce themselves to you
in your first couple of days, but if you want to form lasting bonds with your office mates,
you will have to continue the conversation. It can be difficult to remember the number of
names and faces you learn those first few days, let alone any personal details about them.

5- Let’s say you easily run into a reasonable work-life balance at your old job. If your new
company expects you to be available on slack more often than your old one, you could have
trouble adjusting at first. And if your boss unexpectedly asks you to spend several extra
hours per week working, that could prove troubling too. Bringing your concerns to your
boss can be the first step toward finding a balance that better suits both you and your team.

A) Answer the following questions:


1. What is the most difficult aspects of starting a new job?
……………………………………………………………….…………………………………………………………………………………..
2. What does participating from the very start show about you?
……………………………………………………………….…………………………………………………………………………………..
3. What is the importance of bringing your concerns to your boss?
……………………………………………………………….…………………………………………………………………………………..
4- There is a fine line between confidence in your ideas and arrogance. Using your own
words, explain the meaning of A FINE LINE in the sentence.
……………………………………………………………….…………………………………………………………………………………..
5- Find AN ADVERB in the passage that means 'Without warning or suddenly'
……………………………………………………………….…………………………………………………………………………………..
B) Match the words to the correct definition:

l)
m) arise – overload – collaborate – bond - adjust
Word Definition
to work with someone else for a special purpose.
a close connection joining two or more people.
to change something slightly, especially to make it more correct.
to start to happen or exist.
to give someone more work or problems than they can deal with.
C) Read the following sentences and decide whether they are true, false or not given by
putting tick (√) in the correct place:

The sentence True False Not Given


1- New employees often find themselves filled with tons of information
and details almost immediately.
2- Old employees at the company tend to like the beginner employee.
3- You have to record any notes for the first two weeks at work.
4- It will be fine if your boss asks you to spend extra hours working.
D) What do the highlighted words in the passage refer to?

Word Reference
The word 'others' para.(2) line (1)
The pronoun 'they' para.(3) line (5)
The pronoun 'them' para.(4) line (4)
E) Summarize the qualities of a good employee in two lines.

……………………………………………………………………………………………………………………………………………………
…………………………………………………………………………………………………………………………………………………….
28- Read the passage and answer the following questions:

The process of refining gold


1- Do you wonder how to refine gold? People use different methods and techniques
to refine the precious metal. It is time to check the steps of raw gold discovering and
refining.
2- Refining raw gold starts from discovering the gold by miners. Geologists make use of
specific maps to search for potential areas with gold deposits. They also consider various
natural formations, such as rocks to conclude whether gold can be present under the earth.
After geologists discover a potential gold deposit location, performing several tests, such as
remote sensing, geophysics, and geochemistry are essential to confirm the prediction.
3- Following this, geologists collect rock samples via a drilling method. Next, they start
analyzing the samples. As a result, they can confirm whether gold is present below the
ground in that spot. The next step is about deciding the type of mine. Miners can start the
mining operation once all the preparation tasks get completed. However, proper
infrastructure is essential for mining. It includes roads as well as processing provisions.
Mining in remote areas needs offices and storage provisions.

4- When the location is ready to start the operation, workers take samples and send them to
engineers to find out the accurate metallurgic standards of gold deposits. Gold remains in
its raw form after its extraction from the mine. Further processing is needed to receive pure
gold.

5- The first step is to crush the parent rock. Next, the pieces go through multiple processes.
It depends on the types of minerals and materials combined with the respective gold
mineral. The goal of these steps is to extract the gold from all these elements. However,
complexity differs in different on-site processing operations. It partly depends on the
standard of the gold mineral. The higher the standard of the mineral, the complexity
becomes higher.

6- Once the on-site processing gets completed, people transfer the gold to an off-site
refinery. It is about making it goes through additional refining. The process eliminates the
remaining impurities (if any) from the gold. The raw gold experiences a melting process and
treatment with chloride. As a result, any impurities remaining with gold convert into
chloride, which automatically gets removed from the gold. The result would be 99.5% to
99.99 pure gold. The final step is about reclaiming the gold mine.
Eng. (102) Reflect Final Revision Activities Prepared by: Mr. Wael Bakry

A) Match the heading with the suitable paragraph:


Heading Paragraph number
Steps of extracting gold from deposits
The site is ready
Pure gold is ready
What are the steps?
Getting samples to be examined
Exploring stage
B) Rewrite the following sentences correctly:
1. Discovering raw gold starts from refining the gold by miners.
……………………………………………………………….…………………………………………………………………………………..
2. Geologists analyze rocks. Then, they collect the samples.
……………………………………………………………….…………………………………………………………………………………..
3. Miners can start the mining operation before all the preparation tasks get completed.
……………………………………………………………….…………………………………………………………………………………..
4- Once the off-site processing gets completed, people transfer the gold to an on-site refinery.
……………………………………………………………….…………………………………………………………………………………..
C) Match the words to the correct definition:

n) refine – deposit – confirm – extract - impurity


o)

Word Definition
to prove or say that something is true.
to make something pure, especially by removing unwanted material.
the fact that a substance is dirty or lower in quality as it is mixed with another substance.
to remove or take out something.
a substance or layer that is left, usually after a liquid is removed.
D) What do the highlighted words in the passage refer to?
Word Reference
The word 'they' para.(3) line (1)
The pronoun 'them' para.(4) line (1)
The pronoun 'it' para.(6) line (2)
E) Find words in the passage with the same meaning

Word/phrase Synonym
important para.(2)
process para.(4)
Quality of level para.(5)
29- Read the passage and answer the following questions:

An Overview and History of UNESCO


By Amanda Briney

1- The United Nations Educational Scientific and Cultural Organization (UNESCO) is an


agency within the United Nations that is responsible for promoting peace, social justice,
human rights and international security through international cooperation on educational,
science, and cultural programs. It is based in Paris, France, and has over 50 field offices
located around the world.

2- Today, UNESCO has five major themes to its programs which include; education, natural
sciences, social and human sciences, culture, and communication and information. UNESCO
is also actively working to achieve the United Nations' Millennium Development Goals but it
is focused on achieving the goals of significantly reducing extreme poverty in developing
countries, developing a program for universal primary education in all countries, eliminating
gender inequalities in primary and secondary education, promoting development and
reducing the loss of environmental resources.

3- UNESCO provides technical advice and support to develop the institutional and human
capacity of countries to achieve their education goals. This includes training of education
practitioners and officers in multiple fields, including educational planning, curriculum
design, data collection and distance learning.

4- Natural sciences and the management of Earth's resources is another UNESCO field of
action. It includes protecting water and water quality, the ocean, and promoting science and
engineering technologies to achieve sustainable development in developed and developing
countries, resource management and disaster readiness.

5- Social and human sciences is another UNESCO theme, promotes basic human rights, and
focuses on global issues like fighting discrimination and racism. Culture is another closely
related UNESCO theme that promotes cultural acceptance but also the maintenance of
cultural diversity, as well as the protection of cultural heritage.

6- Finally, communication and information is the last UNESCO theme. It includes the "free
flow of ideas by word and image" to build a worldwide community of shared knowledge
and empower people through access to information and knowledge about different subject
areas.
A) Match the heading with the suitable paragraph:
Heading Paragraph number
Its learning objectives
Knowledge all over the world
Human rights
What is UNESCO?
Safe sources of water
Its objectives
B) Answer the following questions:
1. What is UNESCO in charge of?
……………………………………………………………….…………………………………………………………………………………..
……………………………………………………………….…………………………………………………………………………………..
2. Mention THREE matters of UNESCO?
a) ………………………………………………………………………. b) …………………………………………………………………
3. How can UNESCO achieve educational goals?
……………………………………………………………….…………………………………………………………………………………..
……………………………………………………………….………………………………………………………………………………….. 4-
The pronoun 'their' in paragraph three refers to ……………………...
a) education goal b) technical advice and support c)
countries C) Match the words to the correct definition:

justice – poverty – collection – sustainable – theme - empower


b)
Word Definition
able to be maintained or continued.
fairness in the way people are dealt with.
to give someone official or legal authority, or the confidence to do something.
the main subject of a talk.
the condition of being extremely poor.
a lot of things or people.
D) Read the following sentences and decide whether they are true, false or not given by
putting tick (√) in the correct place:
The sentence True False Not Given
1-The main office of UNESCO is in France.
2- Poverty in developing countries is not the main concern of UNESCO.
3-UNESCO has so many missions in Africa.
4- UNESCO helps poor countries fight natural disasters like volcanoes.
30- Read the passage and answer the following questions:

Advantages and disadvantages of voluntary work

1- Volunteering is a concept of participating in multiple beneficial activities for the good of


fellow humans without reward or remuneration. Volunteers are those who prefer to serve
the other people in varying ways to find the inner peace and quietness. The following essay I
have written that tells you the meaning and concept of volunteering, advantages and
disadvantages of volunteering for students.

2- Volunteering for social causes is the preferred human virtue. Volunteering is the unpaid
work done by people who donate their time for the welfare of others or for some social causes.
This can be in the form of donations of goods, services, or money. Volunteering is often
associated with charitable organizations, but it can also be done independently. People
volunteer for many different reasons such as to help others, to gain new skills, to meet new
people, or to have fun.

3- Volunteering has many benefits for both the individual and the community. For
individuals, volunteering can improve mental and physical health, provide social support,
and increase self-confidence. It can also help people develop new skills and gain work
experience. For communities, volunteering can boost the economy, improve social cohesion,
and increase civic engagement.

4- Despite the many benefits of volunteering, there are some drawbacks to consider as well.
For example, volunteers may not be properly trained or supervised, which can lead to safety
concerns. In addition, volunteering can sometimes be used as a way to exploit workers by
providing them with little or no pay.

5- Students can be a great help to the society by volunteering their time and skills for the
community benefits. They can contribute to the on-going projects or work on new
initiatives as part of their curriculum requirements. They can teach other community kids,
help elders, and work for the betterment of their societies.

6- The benefits for the students are plenty. They learn about discipline, teamwork, and
leadership skills by working on projects together. They learn to appreciate the value of time
and how to use it in a constructive way. Volunteering can also help them develop new skills
and interests, and build their resume.
A) Match the heading with the suitable paragraph:

Heading Paragraph number


The advantages
What is voluntary work?
It is useful for students
Pupils can help
For free
The negatives
B) Answer the following questions:
1. Who are volunteers?
……………………………………………………………….…………………………………………………………………………………..
2. What is Volunteering is often linked with?
……………………………………………………………….…………………………………………………………………………………..
3. Volunteering has many merits for communities. Mention TWO of them.
……………………………………………………………….…………………………………………………………………………………..
4- "Students can help community". Explain
……………………………………………………………….…………………………………………………………………………………..
……………………………………………………………….…………………………………………………………………………………..
C) Match the words to the correct definition:
c) beneficial – donation – boost – drawback – initiative - appreciate
d)

Word Definition
helpful, useful, or good
something of value such as money or goods that is given to help a person or
organization such as a charity.
to increase or improve something.
a disadvantage or the negative part of a situation.
a new plan or action to improve something or solve a problem.
to recognize how good someone or something is and to value them or it.
D) What do the highlighted words in the passage refer to?
Word Reference
The pronoun 'it' para.(2) line (4)
The word 'both' para.(3) line (1)
The pronoun 'them' para.(4) line (4)
The pronoun 'their' para.(5) line (4)
31- Read the passage and answer the following questions:

Natural and Man-Made Disasters

A natural disaster is a disaster caused by nature, and men have no control over them.
Earthquakes, tsunamis, floods, landslides, hurricanes, wildfires, droughts, volcanic eruptions
are some examples of natural disasters. Such disasters cause massive loss of life, property,
and many other glooms. Although natural disasters are caused by nature, not by humans,
the activities of humans can have an influence on them. For instance, actions of men like
cutting down trees and destroying the sources of water in an area can lead to droughts and
wildfire.

Floods are one of the most common natural disasters that occur in many regions of the
world every year. Flood can be defined as a rising and overflowing of a body of water onto
normally dry land. Heavy rainfall in a short duration of time can result in a flood. Although
the loss of lives in a flood may be not as high as a tsunami or earthquake, floods result in
many long- term problems. Damage to human properties (houses, roads, bridges, power
lines, etc.), shortage of food and drinking water, destruction of forests and animals, the
spread of diseases, soil erosion are some effects of floods.

Drought is the opposite of flood. It is a long period of time during which there is very little
or no rain. Scarcity of water, extensive crop damage, lack of food and drinking water, soil
degradation and erosion, death of animals and conditions like malnutrition are some effects
of drought.

A man-made disaster is a disaster caused by human beings. Some examples of man-made


disasters include hazardous material spills, explosions, chemical or biological attacks,
nuclear blasts, traffic collisions such as train accidents, plane crashes, etc. Most of these
disasters are in the form accidents (except attacks) and cause deaths, injuries, and loss of
property.

The great Smog of London (1952) is an example of a man-made disaster. This smog resulted
in a period of cold weather combined with an anticyclone and windless conditions; the main
cause for this disaster was severe air-pollution. This caused about 4000 thousand deaths
and other respiratory problems.
A) Match the description with the disaster:

Description Disaster
It is a long period of time during which there is very little or no rain.
A disaster caused by human beings
It is resulted in a period of cold weather combined with an anticyclone and
windless conditions
A rising and overflowing of a body of water onto normally dry land.
A disaster caused by nature, and men have no control over them
B) Answer the following questions:
1. What are the consequences of floods?
……………………………………………………………….…………………………………………………………………………………..
……………………………………………………………….…………………………………………………………………………………..
2. What are the main features of drought?
……………………………………………………………….…………………………………………………………………………………..
……………………………………………………………….…………………………………………………………………………………..
3. Find a noun in the passage that means 'illness'. ………………………………………………………………
4. Find a noun in the passage that means 'hunger'………………………………………………………………
4- Air-pollution is ……………………………….. .
a) a man-made disaster b) a natural disaster c) it can be both
C) Match the words to the correct definition: (There is an extra definition)

e) massive – duration – scarcity – hazardous - smog


f)

Word Definition
when something is not easy to find or get.
air pollution caused by smoke or chemicals mixing with fog.
very large in size, amount, or number.
physical weakness and bad health caused by having too little food.
dangerous and involving risk.
the length of time that something lasts.
D) What do the highlighted words in the passage refer to?

Word Reference
The pronoun 'them' para.(1) line (5)
The word 'This' para.(6) line (3)
E) Suggest two solutions for air pollution problem?

a)………………………………………………………………………………………………………
b)………………………………………………………………………………………………………
32- Read the passage and answer the following questions:

Junk Food
1- Any food with zero or insignificant nutritional value, high in fat and trans fat content, can
be termed junk food. Most fast foods, including candy, soft drinks, bakery products, burgers,
along with salty and oily foods, can be termed junk food. They are high in calories, salts, and
fats, and are usually low in vitamins, minerals, and other nutritional contents. The more we
consume these foods, instead of healthier and nutritional options, the less will be the intake
of essential nutrients and vitamins needed by the body.

2- One of the major harmful effects of junk foods from your favourite fast food restaurants
is obesity and its related health problems. The extreme sugar, calories, and fats present in
junk foods contribute to weight gain. Obesity can lead to many medical issues, such as
diabetes, joint pain, and heart diseases.

3- Junk foods are loaded with sugar and fats that can cause certain chemical reactions in the
brain, affecting its functioning. The body may lose essential nutrients and amino acids by
consuming too much of them. These symptoms can eventually lead to the inability of the
brain to deal with stress and can leave you feeling depressed.

4- A study published in the American Journal of Clinical Nutrition stated that people who ate
junk food performed poorly on mental tests. It concluded that junk foods can decline your
memory and leave you with a weaker brain. These foods may lead to sudden inflammation
in the hippocampus of the brain that is responsible for memory and recognition.

5- Consumption of excess junk food may leave the brain in a dilemma. Excessive sugar
intake can cause blood sugar levels to fluctuate and make the brain demand more food,
eventually leading to overeating. This can make it difficult for the body to digest excessive
junk food.

6- The lack of nutrients and vitamins is one of the many bad effects of junk food. Nutrients
and vitamins are essential for the proper growth and development of the body. Unhealthy
eating habits, even if they involve consuming junk food in small amounts, combined with an
imbalance of fatty acids and the need for nutrients, can hamper the development of your
brain and other body parts. Excess soda and sugar intake can also lead to tooth decay and
the weakening of bones.
A) Match the heading with the suitable paragraph:

Heading Paragraph number


Depression
Worsens appetite and digestion
What is junk food?
Insufficient growing and development
Loss of memory and learning problems
Obesity
B) Answer the following questions:
1. What is junk food?
…………………………………………………………………………………………………………………………………………………………………
2. Mention THREE diseases caused by eating junk food.
a) …………………………………………. b) …………………………………… c) …………………………….
3. What is the result of the study published in the American Journal of Clinical Nutrition?
…………………………………………………………………………………………………………………………………………………………………
4- 'Consumption of excess junk food may leave the brain in a dilemma.' Give reasons from the
passage.
…………………………………………………………………………………………………………………………………………………………………
…………………………………………………………………………………………………………………………………………………………………
5- The pronoun 'them' in paragraph (3) line (3) refers to ………………………………………………
C) Match the words to the correct definition:

g)Word Definition
h)
nutrient – symptom– decline – overeating
any feeling of illness or physical or mental change - lackby a disease.
that is caused
the fact that something is not available or that there is not enough of it.
any substance that plants or animals need in order to live and grow.
the action of eating more food than your body needs.
to gradually become less, worse, or lower.
D) Find words in the passage with the same meaning
Word/phrase Synonym
Not important para.(1)
weakness para.(3)
33- Read the passage and answer the following questions:
Reasons why Martial Arts is better than team sports
The number one reason why most people decide to study Martial Arts is it is a great way to
learn how to defend yourself. Martial arts do not only teach you the techniques on how to
defend yourself, but also gives you the mental preparation as well. By training you will learn
how to expect and avoid potential dangers. As children become more talented in their training,
their confidence gets a big-boost. They become more self-assured and confident not just in
their Martial Arts skills, but in other life skills as well.
Martial Arts training is a physical expression of practice makes perfect. In a class, a student may
execute a single movement or series of related movements, hundreds of times. This teaches the
student that the progress in their training will come through endless repetition. Many kids have
the chance to experience the benefits of Martial Arts early on, however a big number of children
abandon their martial arts training when reaching the teen years in order to participate in
school sports.
I am sure that while many parents would consider martial arts for their sons, not nearly as many
would consider it for their daughters. However, martial arts are one of the few sports where
both boys and girls can play together. We have a large amount of girls in our school and in
many classes they outnumber the boys. The girls are among the most technically proficient
students we have.
Childhood obesity is a global epidemic and rising trends in overweight and obesity are obvious
in both developed and developing countries. The typical martial arts class will often be involved
in teaching and practicing of moves and possibly some fighting, as well as some active games.
The warm up and practice include the bulk of the time.
Martial Arts are meant to be practiced with the mindset that your opponent will be bigger and
stronger than you. That technique and mental toughness should be pursued above all physical
attributes.
Martial Arts is trained by many people all over the world. It is very common to see a school no
matter where you may be in your travels. The reason for this is the global understanding of the
benefits of training. Remember not all schools and instructors are created equally. Make sure to
do proper research before choosing a school that is right for you.
A) Match the heading with the suitable paragraph:

Heading Paragraph number


Discipline and individual Achievement
Martial Arts isn’t about brute force, it is about control
Martial Arts is World Wide
Gender Equality
Self-Defense and Self-Confidence
Weight Control
B) Answer the following questions:
1- The first reason why most people decide to study Martial Arts is ……………… .
a) weight control b) gender equality c) defend yourself
2- "Practice makes perfect" means ……………… .
a) progress in training comes through repetition b) single movement is important
c) defending yourself is important
3- The number of girls in our school and in many classes is..........................boys.
a) as equal as b) more than c) less than
4- The underlined word 'execute' in paragraph (2) line (2) means ………………………
a) do b) murder c) help
C) Match the words to the correct definition:

i)
j)
self-assured – abandon – proficient – obvious - toughness

Word Definition
to leave a place, thing, or person, usually for ever.
easy to see, recognize, or understand.
the quality of being not easily defeated.
skilled and experienced.
having confidence in your own abilities.
D) Read the following sentences and decide whether they are true, false or not given by
putting tick (√) in the correct place:

The sentence True False NG


1- Martial Arts teach children to become confident in skills only. 
2- Karate and taekwondo are male martial arts. 
3- Overweight and obesity are clear in both developed and developing countries. 
4- All schools and instructors of Martial Arts are equal. 
E) What do the highlighted words in the passage refer to?

Word Reference
The pronoun 'their' para.(1) line (5)
The pronoun 'their' para.(3) line (2)
F) What are the advantages and disadvantages of Martial Arts?

Advantages:……………………………………………………………………………………………………
……………………………………………………………………………………………………………………
Disadvantages:…………………………………………………………………………………………………
……………………………………………………………………………………………………………………
34- Read the passage and answer the following questions:

How Does Football Make Money?


From the ground up, literally from the pitch and stadium it is played in, everything in football
is monetized in some way to generate revenue. When players like Ronaldo are making tens
of millions of euros every year, you know there is a serious incentive to keep them on the
field and performing highly entertaining sports. At the end of the day, this is precisely what
sports are, entertainment. Let us look at some of the biggest factors to the economy of
football.
Stadium
Traditionally, stadiums in countries like England were named after the cities the stadiums
were founded in. After generations of football's viewer base increasing dramatically, big
companies have found that it is great experience to gain naming rights to these iconic
stadiums. In 2004, the team Arsenal signed an exclusive deal with the airline Emirates for
naming rights on their stadium, paying £100 million for 15 years. This is certainly a lucrative
proposal for any club and indeed, all major sports teams follow this trend and as a result, the
company's name is mentioned every game and is in front of millions of viewers.
TV Deals
By far the biggest moneymaker for teams around the world is the television distribution
deals which are essentially the most important revenue source for every league. In 2016, the
Premier League signed its record-breaking deal for £10.4 billion distributed amongst the
league's clubs. Out of that sum, £5.3 billion was for the broadcast of live matches and
another £5.1 billion was for sale of overseas rights. An impressive deal to say the least and
there are sure to be bigger deals in the future.
Sponsorships
Every football player is essentially a walking advertisement for the numerous businesses that
sign deals with the clubs. Naturally, businesses can see the value in these sponsorships and
invest huge sums into winning teams. Every club have their own sponsorships and make
large contributions the clubs can use to get better players. All football jerseys are printed
with several sponsors much in the way a sport like NASCAR racing has several sponsorships
on their racecars.
Attendance and Events
Like television deals, crowd attendance is one of the fundamental and most important
aspects of football's annual revenue. A club like Real Madrid is estimated to make over €4.9
million per match, accounting for up to 23% of their total annual revenue. This adds up to
an immense €131 million per year, an amazing amount for any major league sports team.
A) Match the sentence with the factor of economy: (There is one extra idea)

Idea Factor of economy


Every team must have one to get better players
Helps in players injury
Takes the name of the city
It is the main source of income to any club
All the teams got money from
B) Match the words to the correct definition:

monetize – lucrative – distribute – jersey - fundamental

Word Definition
producing much money or making a large profit.
more important than anything else.
a shirt that is worn by a member of a sports team.
to change something into money.
to give something out to several people, or to spread or supply something.
C) Read the following sentences and decide whether they are true, false or not given by
putting tick (√) in the correct place:

The sentence True False NG


1- Millions of euros every year were great incentive for Ronaldo.
2- The name of the sponsor is printed on the shirts of players.
3- Audience is not an important source of money in football.
4- One of the most famous sponsors on Real Madrid t-shirt Lufthansa.
D) What do the highlighted words in the passage refer to?
Word Reference
The pronoun 'them' para.(1) line (3)
The pronoun 'their' para.(2) line (4)
The pronoun 'its' para.(3) line (3)
The pronoun 'their' para.(4) line (6)
The pronoun 'their' para.(5) line (3)
E) Find words in the passage with the same meaning:

Word/phrase Synonym
established para.(2)
many para.(4)
35- Read the passage and answer the following questions:

The Importance of Sport for The Children’s Health

1- Sport helps children to develop both physically and mentally. Physical activity
should be part of our daily life. Practicing any kind of sport from an early age helps
children in many ways, for example, group activities improve interaction and social
integration. It also improves the mental level and promotes a healthy lifestyle in
adulthood, as the habits we acquire as children are often to stay. Sports have multiple
healthy benefits. If we talk on a psychological or personality level, sports activities
(especially those done in groups) help children to strengthen their social self-esteem,
and allow them to understand the importance of having and respecting the rules.

2- Another aspect to consider is what type of sport or activity will suit best the child
considering his/her conditions and abilities. A more timid and shy child is likely to get
better at a sport where she/he has to relate to peers. To a more active one, an
individual sport such as tennis can help better develop her/his ability to concentrate.
One of the perhaps most relevant issues is the importance of physical exercise, not
only in children but also in young and old people, because moderate daily physical
exercise helps prevent overweight and obesity. Taking into account the growth of the
percentage of children and young people with obesity and overweight in the Western
world, the inclusion of sport in the family routine becomes almost mandatory.

3- Sport can also have the power to help us rest. For all those fathers and mothers of
active children, try to get them to run every day playing soccer, or swimming or any
other activity that keeps them physically active. They will fall asleep in bed in a matter
of seconds and you, dads and moms, too.

A) Answer the following questions:

1. How are group activities helpful for children?


…………………………………………………………………………………………………………………………………………………………………
…………………………………………………………………………………………………………………………………………………………………
2. How do psychological and personality sports activities help children?
…………………………………………………………………………………………………………………………………………………………………
…………………………………………………………………………………………………………………………………………………………………
3. What are the most favorable sports for both shy and active children?
Shy child: …………………………………………….. Active child: …………………………………………………..
4. Physical exercise is important for all ages. Give reason from the passage.
…………………………………………………………………………………………………………………………………………………………………
5.How is sport helpful to parents?
…………………………………………………………………………………………………………………………………………………………………
B) Match the words to the correct definition: (There is an extra definition)

k) interaction – strengthen – aspect – prevent - soccer


l)

Word Definition
to make something stronger or more effective.
a situation where two or more people communicate with each other.
a particular feature of or way of thinking about something.
must be done, or is demanded by law
to stop something from happening
a game played between two teams of eleven people, where each team tries
to win by kicking a ball into the other team's goal.
C) Read the following sentences and decide whether they are true, false or not given by
putting tick (√) in the correct place:

The sentence True False Not Given


1- Mental activities are those related to mind such as reading,
listening and watching TV shows to are important.
2- Sports have several healthy benefits.
3- Sport is not required in the family routine .
4- Both parents and kids need to sleep after physical activities.
D) What do the highlighted words in the passage refer to?

Word Reference
The pronoun 'those' para.(1) line (6)
The word 'one' para.(2) line (3)
The pronoun 'them' para.(3) line (2)
Part Two: Use of Language (Vocabulary)
A) Complete the sentences with the correct form of the words in the box
1- Complete the sentences with the correct form of the words in the box.
influence – admire – debate – original – offend

1. If there is a.........................about something, people express different opinions about it.


2. If you...............................something, you think it is beautiful or special.
3. Something that is......................existed or happened first, before any others.
4. If you...................someone, you make that person upset by doing something they think is
unacceptable.
5. A person who has a great......................on you affects how you think or behave.
2- Complete the sentences with the correct form of the words in the box.
community – powerful – rural – surrounded – privacy

1. My house is................................by trees. It’s like living in a park!


1. You can become part of an online............................and meet people with similar interests.
2. It’s important to respect people’s..................and not divulge sensitive information about them.
3. My cousin lives in the countryside. She prefers.....................areas instead of the city.
4. The doctor gave me some............................medicine to help me cure this bad cough.
3- Complete the sentences with the correct form of the words in the box.
performance – physical – obvious – access – former

1. It is.......................that most teenagers these days enjoy using smartphones.


2. When children don’t sleep enough, it can affect their.............................at school.
3. Marilyn is a …………………… teacher--she became a nurse two years ago.
4. People often use computers and phones to get..........................to the Internet.
5. Everyone needs to do some..........................activity to be healthy.
4- Complete the sentences with the correct form of the words in the box.
solve – experiment – hire – aim – think of

1. When companies want to........................someone, they often advertise the job online.
2. For many people, the first iPhone..................the problem of needing to carry a phone, a music
player, and a camera separately.
3. When you.....................a good idea, write it down before you forget it.
4. Most companies......................to increase sales from year to year.
5. Ice-cream companies often....................with new flavors to increase sales.
5- Complete the sentences with the correct form of the words in the box.
absolutely – ahead – behavior – conscious – painful

1. Please try to be kind. If you hurt people’s feelings, it can be...........................for them
2. Make sure you take regular breaks when you have a long day of studying...................of you.
3. I don’t know why I just laughed. I didn’t have a.......................thought about something funny;
it just happened!
4. There is.................no doubt that positive memories can help people to feel happier.
5. Even when Ali feels stressed, he doesn’t act differently. His...........................is always calm.
6- Complete the sentences with the correct form of the words in the box.
particular – stereotype – meaning – cases - valuable

1. A............................is a belief, often untrue, about what a certain type of person or thing is like.
2. Something that is.....................is important or precious, often because it is rare.
3. If there are several..................of something, that means there are several examples of it.
4. The..........................of something is the quality that gives it real value and importance.
5. A.........................person or thing is a specific one, apart from any other.
7- Complete the sentences with the correct form of the words in the box.
virtual – trends – urban – protect – paradox

1. The city council is planning to build more parks in............areas to add some green in the cities.
2. It’s a..........................that big cities are very crowded yet many people feel very lonely.
3. I have over 200.......................friends on my social network.
4. Eating healthy food and working out can help to.....................you from getting some illnesses.
5. My sister loves clothes, so she keeps up-to-date with all the latest..........................in fashion.
8- Complete the sentences with the correct form of the words in the box.
acquire – intelligence – preferable – invent – development

1. Scientists should...................a machine that will let people travel back in time.
2. For most jobs, you need to.......................some computer skills
3. In the early stages of their........................, children learn to walk and talk.
4. People use their.........................to figure out answers to problems.
5. For many children, playing with friends is.........................to doing homework.
9- Complete the sentences with the correct form of the words in the box.
value – analyze – data – complain – employee

1. It is important to..........................customer needs before opening a new business.


2. My boss............................creativity in the people she works with
3. at Google get free food and gym classes.
4. Your smartphone keeps a lot of.....................about you, such as the websites you’ve visited.
5. When customers................about something that makes them unhappy, it is often best to just
listen to them.
10- Complete the sentences with the correct form of the words in the box.
personality – protect – psychology – relationship – situation

1. My uncle is a professor of....................., so I like talking to him about my thoughts and feelings,
which he helps me to understand.
2. My baby sister definitely knows what she wants! It’s already clear that she has a strong …………
3. When you take driving lessons, you also learn how to manage any dangerous ……………………..
on the road.
4. Eating healthy food and working out can help to....................you from getting some illnesses.
5. The two brothers didn’t get along when they were children, but they have an excellent
………………….. now
11- Complete the sentences with the correct form of the words in the box.
version - predictable - pronounce - suitable - expression

1. Are there different....................of your language spoken by different groups and communities?
2. What are the most difficult words to....................in your language? In English, they’re
words with lots of consonants together, such as strengths.
3. What’s your favorite.....................in your language? In English, mine is “It’s only money”.
4. Do you sometimes find it difficult to think of a...................word or phrase to describe a feeling?
5. English spelling is not very........................complicated: you can’t say how a word will be
spelled just from hearing it. Is it the same in your language?
12- Complete the sentences with the correct form of the words in the box.
surroundings – aspect – industrial – suburb – associate

1. We live in a.............................of London.


2. Everyone wants safe, comfortable ………………….
3. Menaa Salman is a big.......................city.
4. I always...........................the smell of baking with my childhood.
5. Have you thought about the problem from every..........................?
13- Complete the sentences with the correct form of the words in the box.
compete – accompany – look into – transport – cooperative

1. I need to.................................the effects of video gaming for my project, so I’m going to learn
how to play a few.
2. At video game conventions, gamers.....................against each other for prizes.
3. Everyone in our group was........................, so we finished the project without any problems.
4. We need a big truck to..............................all of the company’s computer equipment.
5. If you don’t want to go to the computer repair store alone, I can..........................you.
14- Complete the sentences with the correct form of the words in the box.
restrict – period – device – usage – shocking

1. Parts of the motor wore out because of constant ………………………..


2. Having small children really..............................your social life.
3. There are few crimes more truly..............................than the murder of children.
4. The study will be carried out over a six-month ……………………………..
5. He invented a........................for measuring very small distances exactly.
15- Complete the sentences with the correct form of the words in the box.
awful – individual – pleasure – symptom– depression

1. We had an.............................training with our coach yesterday.


2. The doctor told us some information about the ……………… of covid-19.
3. Our experience with this company was....................one. They provide very bad service.
4. I’m just beginning to get over the.......................from losing my job.
5. The boy’s visits gave his grandparents a great deal of................They were extremely delighted.
16- Complete the sentences with the correct form of the words in the box.
logical - translate - complicated - Similarly - replace

1. Is the grammar of your language........................., or is it easy to learn?


2. Saying “I lucked out” to mean “I was lucky” doesn’t seem....................Are there
examples in your language of things that don’t seem to make sense?
3. Are there any cases where an English word has...............the word usually used in your language?
4. In French, all nouns are either masculine or feminine....................., nouns in Italian
always have a gender, too. What about in your language?
5. Can you think of a word in your language that you can’t easily....................into English?
17- Complete the sentences with the correct form of the words in the box.
attachment – distinctive – proud – quality of life – historical

1. She's got a very.............................voice.


2. is one of the most important elements of happiness for citizens.
3. Many important.....................documents were destroyed when the library was burnt.
4. She had a special............................to these students.
5. You must be very...............................of your son.
18- Complete the sentences with the correct form of the words in the box.
remain – audience – animation – approximately

1. Some games are created for a specific...................., such as people who love sports.
2. According to a recent study,.................35 percent of video game players are 21 to 35 years old.
3. Many people...........................in their hometowns their whole lives.
4. There are many free software programs that help you create …………………………….
19- Complete the sentences with the correct form of the words in the box.
recover – mood – experience – technique – experiment

1. Don't forget to try out the equipment before setting up the ……………………………...
2. It took him a while to................................after the operation.
3. Yoga is a very effective...................................for combating stress.
4. Our teacher is in a good...............................today. She is smiling all the time.
5. I had a pretty unpleasant.............................at the dentist's. My tooth was bleeding so much.
20- Complete the sentences with the correct form of the words in the box.
look at – substantial – notice – challenge

1. That is a very...............................improvement in the present situation.


2. He played a tricky piece on the piano and it was a big..........................for me to do the same.
3. I waved at my friend but he didn't seem to ……………...
4. Management is..........................ways of cutting costs.
21- Complete the sentences with the correct form of the words in the box.
beneficial – boost – flexible – mental – bilingual

1. A doctor was asked about the.........................state of the prisoner.


2. From our point of view, we do not see how these changes will be.....................to the company.
3. My schedule is...................... - I could arrange to meet with you any day next week.
4. He is........................in Arabic and English.
5. The company is looking for ways to........................sales in Asia, its biggest market.
22- Complete the sentences with the correct form of the words in the box.

accomplish – warn – open to – multicultural – function

1. The country has a rich.........................heritage.


2. He.....................such a lot during his visit last week.
3. One of your..........................as receptionist is to answer the phone.
4. Have you........................them that there will be an extra person for dinner?
5. he competition is.........................anyone over the age of 16.
23- Complete the sentences with the correct form of the words in the box.
commonsense – trade – respect – sensitive – phenomenon - brand

1. What is your favorite.....................of shampoo?


2. When you visit another country, be sure to...................the local customs.
3. Some people laughed when I gave the wrong answer, but Tom didn’t. He’s very........................!
4. I am sure that...................will prevail in the end.
5. Our company has plans to.............................with businesses in Africa.
6. There's evidence to suggest that internet addiction is not just a recent ………………………..
24- Complete the sentences with the correct form of the words in the box.
globalization – impression – guideline – profit – found

1. It takes a lot of money and effort to............................a successful business.


2. The.......................on personal use of the Internet in the office is clear: it is not allowed.
3. It’s important to make a good first.....................at a job interview.
4. The financial situation was bad, but some companies were able to......................from it.
5. has resulted in the loss of some individual cultural identities.
25- Complete the sentences with the correct form of the words in the box.
argue – strategy – material – look up – nearby

1. If you want to improve your creativity, one...............................is to go on long walks.


2. Keep your................................dictionary when you read a book in a foreign language.
3. Scientists........................that people shouldn’t have much sugar because it can be unhealthy.
4. You can..............................the information you need online.
5. If you need more..............................to write your paper, you should read this book.
26- Complete the sentences with the correct form of the words in the box.
relevant – disturbing – convenient – combination – tend

1. It’s very..........................for me to shop here because I live only one block away.
2. This article about rivers is not.......................to my essay about trees.
3. Children..........................to enjoy playing outside in the snow.
4. This soup is a.......................of rice, chicken, and vegetables.
5. Please turn off the TV while I’m working—the noise is very ……………………...
27- Complete the sentences with the correct form of the words in the box.
effective – conscious – actively – work on – Otherwise

1. Most people agree that you cannot control your dreams with your....................brain.
2. All the employees had to.........................the problem together in order to solve it.
3. Raul is a very....................manager because he communicates clearly with his employees.
4. Go to bed early tonight....................., you might be late for your interview tomorrow morning.
5. Hana is trying............................to improve her English by taking an English class.
B) Choose the correct answer:
1. Mr. Ito knows a lot about the benefits of boredom. He’s done a...............amount of research on it.
a) restrict b) substantial c) shocking
2. I’m trying to..................how much money I spend every month. I need to save more.
a) delete b) restrict c) hire
3. Between 2011 and 2017, Internet..................almost doubled worldwide.
a) usage b) challenge c) delete
4. Manar wears a fitness......................that records her physical activity and other data.
a) device b) challenge c) experiment
5. I only saw the thief briefly. He was tall with brown hair, but I didn’t...............the color of his eyes.
a) delete b) notice c) restrict
6. When you face a.........................., try not to give up. Just do your best!
a) notice b) usage c) challenge
7. We couldn’t believe how rude those people were. Their behavior was absolutely....................!
a) appropriate b) substantial c) shocking
8. I don’t think I agree with that scientist’s theory. I’m going to...............more research on the topic.
a) look at b) restrict c) delete
9. My computer is running out of memory. I’ll have to..................some files to save space.
a) notice b) delete c) hire
10. Gabriel only started playing the piano last month. He’s improved so much in a short..............of time.
a) period b) usage c) period
11. I decorated my jeans with colorful ………………………. .
a) patches b) diet c) vehicles
12. They....................the elections to make sure everything is fair.
a) monitor b) diet c) detect
13. A popular children’s game is cops and ……………………… .
a) patches b) restrict c) robbers
14. Many people think we won’t need vehicles and we’ll only travel via.................in the future.
a) teleportation b) diet c) robbers
15. Please calm down. There’s no point getting ……………………………...
a) disabled b) monitored c) agitated
16. The driver lost control and hit an...........................truck.
a) oncoming b) agitated c) restricted
17. This alarm can.......................smoke from a cigarette.
a) burn b) transcend c) detect
18. There is a...............................parking facility.
a) disabled b) detected c) agitated
19. I read an article about how reducing the amount of sugar in one’s...............can be really beneficial.
a) diet b) monitored c) patches
20. This music is so beautiful that it.......................words and language.
a) transcends b) monitors c) agitated
21. Please...................any irrelevant information from the document before submitting it.
a) delete b) restrict c) hire
22. The experiment aims to................the effects of different factors on plant growth.
a) transcend b) experiment c) detect
23. The company decided to.....................the use of personal devices during working hours.
a) stay in touch b) teleportation c) restrict
24. She felt.....................when she realized her phone was missing.
a) shocking b) agitated c) complain
25. Many people have reported serious issues with this computer battery. It seems to be a ………………
problem.
a) common b) trivial c) dreadful
26. The children are so happy playing games in the park. There’s a wonderful..............of joy about
them!
a) trend b) sense c) disadvantage
27. She’s my favorite author, so I always buy her books as soon as they ………………………..
a) cooperate b) come out c) ignore
28. He used to be nervous and shy, but he developed a lot of..............................., and now he’s an
excellent public speaker.
a) disadvantages b) cooperation c) confidence
29. I spend so much time on my phone every day. I think I might be............................to it!
a) common b) addicted c) trivial
30. This video game has a cool historical theme. But you’ll still enjoy it...........you’re interested in
history or not.
a) whether b) sense c) confidence
31. My uncle sent me money to buy new headphones. He said it was only...........amount but, actually, it
was a lot!
a) addicted b) common c) trivial
32. There are many good points about living in New York City, but the main..................is that it’s
extremely expensive.
a) disadvantage b) advantage c) sense
33. My colleague and I have very different personalities. But even when we disagree, we ……………………
at work and our projects are successful.
a) ignore b) cooperate c) fight
34. My children always answer my texts, but when I call, they.....................me.
a) look out b) come out c) ignore
35. If you accompany someone, you...........................them.
a) go with b) call c) remember
36. Approximately 25 percent means......................25 percent.
a) exactly b) about c) more than
37. An audience is a group of people who ……………………………..
a) work together b) play a sport c) watch or use something
38. When you compete in an activity, you try to ……………………………..
a) have fun b) win c) meet people
39. When you make an animation, you make ……………………..
a) a movie with real people b) a photo c) a drawing that moves
40. When you are cooperative, you are...........................to work with.
a) easy b) fun c) interesting
41. If you look into something, you................................it.
a) find out about b) explain c) create
42. If you remain in a place, you …………………………………...
a) move there b) stay there c) explore
43. If you transport something, you............................it.
a) manage b) buy c) move

Part Two: Use of Language (Grammar)


A) Choose the correct answer:
1. If you listened carefully, you............................realize that he’s upset.
a) would b) can c) have
2. We.............................complain if we were responsible for the failure.
a) couldn’t b) haven’t c) didn’t
3. If you showed her a photo, it...........................her to remember.
a) did help b) would help c) is helping
4. I...................................any of this if it wasn’t completely true.
a) won’t say b) didn’t say c) wouldn’t say
5. If I forgot an important detail, I might...........................the test.
a) failed b) failing c) fail
6. Today, she is ............ her grandparents, she loves...........with her grandmother.
a) visiting, cook b) visit, cooking c) visiting, cooking
7. Her grandma usually teaches her how to cook delicious dishes. At this moment, Indri is ………..
how to make tasty chicken soup.
a) learn b) learned c) learning.
8. Which sentence uses a Present Participle in a gerund?
a) I enjoy writing. b) The broken toy was in the trash. c) She is singing a lovely song.
9 with a glass of water, these tablets will cure a headache.
a) Taking b) Taken c) Takening
10 a fan of Sherlock Holmes, I have all the Conan Doyle novels.
a) Been b) being c) be
11 meet new people, she preferred to stay at home.
a) In order to b) In order not to c) So.
12. Hadi plays tennis three times a week..........................improve his game.
a) because b) to c) in order not to.
13. In order to save money, Ali is going to buy some of his needs only.
a) In order to b) In order not to c) for
14. Fatima spent all week studying..............................fail her exam.
a) in order not b) in order to c) in order not to
15. show he wasn’t angry, Sameh didn’t say anything about the argument.
a) In order b) To c) Because
16. She....................................in Florida but prefers California.
a) live b) living c) lives
17. Where.........................she live?
a) does b) do c) is
18. Soha...........................every evening.
a) washes up b) wash ups c) wash up
19. My parents............................work on Saturdays.
a) isn't b) doesn't c) don't
20. What time..........................your father go to work?
a) is b) does c) do
21. If you had let me know earlier, I...................have been able to come.
a) would b) will c) shall.
22. You......................help me yesterday.
a) must b) should to c) had to.
23. If there is an age restriction then you go in as you are too young.
a) had to b) shouldn't c) might
24. She is so different from you, Claire. You....................be her sister.
a) has to b) must c) can’t
25. I'd like to buy her this black-and-white dress, but she.......................like it, I'm afraid.
a) cannot b) should not c) may not
26. They could argue all day if they..........................enough time.
a) had b) have c) has
27. I might not............................................his name if he didn’t remind me of it frequently.
a) remembered b) remembering c) remember
28. If you............................enjoy that documentary, you might like this movie.
a) enjoyed b) will enjoy c) would enjoy
29. I would forget so many things if I.............................write them in my calendar.
a) don’t write b) didn’t write c) can’t write
30. If she...........................advice, she could ask her manager for suggestions.
a) would need b) will need c) needed
31. After.........................breakfast, they went out for a walk.
a) finishing b) finished c) the finishing
32. While........................tennis, Sara fell and hurt her ankle.
a) play b) playsing c) playing
33.................................online, plane tickets are usually cheaper.
a) Purchased b) Purchasing c) purchase
34. The market was full of tourists........................souvenirs.
a) buy b) buyers c) buying
35. That lady...........................on the phone must be the manager.
a) talking b) talk c) was talking
36. They never......................Maths.
a) studies b) study c) studying
37. you feel happy?
a) Are b) Does c) Do
38. Abdullah...........................TV after school.
a) watches b) is watch c) watch
39. He.........................do his homework every day.
a) doesn't b) don't c) isn't
40. When does he.............................his room?
a) tidies b) tidy c) does tidy
41. Don't be late for your English exam. You.................be at school on time.
a) must b) may c) shouldn’t
42. You.............watch TV as long as you like. You need to do your homework.
a) can b) must c) can't.
43. You have just eaten. You......................be hungry.
a) can't b) should c) must.
44. Andrew....................eat five slices of pizza in just one meal.
a) must b) have to c) can
45. Be polite! You.............speak rudely to your brother.
a) have to b) shouldn’t c) can
46. What would you......................if you won the lottery?
a) do b) did c) was doing
47. She wouldn't be nervous if she.......................her homework.
a) would do b) did c) does
48. If my grandfather …………. alive, he...................110 years old.
a) would be / were b) was / will be c) were / would be
49. If you printed on both sides, you......................paper.
a) were saved b) would save c) would be save
50. We......................football if the weather were good.
a) would be able to play b) would were able to play c) were able to play
51. My company’s employees are extremely.........................people.
a) profession b) professional c) professionist
52. Our group project was very ………………………….
a) successful b) succession c) successive
53. Experts suggest a...........................mindset to develop problem-solving skills.
a) flexibility b) flexibilic c) flexible
54. To stay.........................., some people need to change jobs every few years.
a) creation b) creative c) creational
55. If I avoid thinking about.......................situations, the solution often comes to me.
a) problemful b) problem c) problematic
56. I appreciated my father’s advice.....................I foolishly didn't follow it the first year I was in
America.
a) while b) even though c) so
57. U.S. team captain Clint Dempsey had a broken nose,......................playing in the World Cup.
a) while b) though c) however
58 Samsung’s profits are improving; Sony’s are going down.
a) Despite b) but c) Though
59 being rich, he is rather unhappy.
a) Although b) Despite c) because of
60 he got up late, he arrived in time for the interview.
a) Despite b) However c) Although
61- Some teachers............................much homework.
a) give b) gives c) are give
62- Taylor and Adam..........................good friends.
a) do b) is c) are
63........................your parents let you go to bed late?
a) Are b) Do c) Does
64- How many books.........................you read in a year?
a) do b) does c) are
65- She ………………. like playing tennis. So she.......................plays tennis.
a) doesn't / always b) doesn't / never c) isn't / never
66. Kate is ill, so she................stay in bed and drink a lot of water.
a) should b) can’t c) have to
67. What do you think? Where..................we go on holiday - New York or Columbia?
a) need b) must c) should
68. You.....................exceed the speed limit. It's dangerous.
a) need to b) mustn’t c) should
69. You.............learn English in this school. There are other languages to choose.
a) don't have to b) should c) mustn’t
70. If it isn't a rule, you.................wear a school uniform.
a) have to b) don't have to c) mustn't
71. This problem...........................by your brother yesterday.
a) solved b) is solved c) was solved
72. This job.................................by my friend next week.
a) will be done b) is done c) was done
73. Many things............................in this house.
a) said b) are saying c) are said
74. The jar............................by the maid.
a) has broken b) has been broken c) broke
75. This car................................by Johnson in the race tomorrow.
a) will be driven b) will drive c) drives
76. The final match...............................at Wankhede Stadium now.
a) is driving b) is being driven c) is driven
77. Students...............................for tricking the class teacher.
a) are being punished b) was punished c) punished
78. Her work.................................by her.
a) were done b) are being done c) has been done
79. Change the statement to passive. 'I am learning yoga.'
a) Yoga is being learnt by me. b) Yoga is learnt by me. c) Yoga was being learnt by me.
80. Change the statement to passive. “I saw a snake.”
a) A snake is seen by me. b) A snake had seen by me. c) A snake was seen by us.
81- One of my friends..........................reads newspaper.
a) doesn't b) never c) isn't
82- How many students.........................English?
a) knows b) does know c) know
83..........................I love spicy food, I hardly ever eat it.
a) Although b) However c) Because
84. The children loved the movie................................it was scary.
a) so b) despite c) though
85 laptops can be expensive, they are very useful.
a) While b) in spite of c) Because of
86 it was very cold, we had a picnic in the park.
a) despite b) Even though c) so
87. He could pass the test...........................answering badly.
a) Despite b) Even though c) While
88 Farrah wanted to use her cell phone, she couldn’t because it didn’t
have any battery.
a) In order to b) Even though c) Despite
89 researching information for his project, Ali went to the library.
a) While b) Because c) Even though
90 smartphones can be distracting, they also offer valuable tools for
learning when used effectively.
a) Despite b) While c) Even though
91. I found the homework difficult.....................................I understood the theory.
a) despite b) while c) even though
92. We spent all day in the library although / in order to complete our class project.
a) although b) while c) in order to
93. The group discussed.............................ways to save money.
a) differing b) different c) difference
94. When you brainstorm, keep writing ideas even if they don’t seem ………………..
a) useful b) useless c) usement
95. I like studying early in the morning. That’s when I’m the most …………………….
a) producting b) product c) productive
96. Employers hope to hire workers with.............................ideas.
a) originate b) original c) originist
97. Practice using facts and statistics in your writing to make it more …………………… .
a) interested b) interesting c) interest
98. If you.......................a bike you wouldn't cycle to school.
a) didn't have b) not had c) weren't have
99. The film …………….. more interesting if it.....................a happy ending.
a) were / would have b) was / would have c) would be / had
100. If my father ………………….. me a ticket, I..............................to the cinema.
a) didn't buy / wouldn't go b) wouldn't buy / didn't go c) wasn't buy / won't go
101. What would you do if you..................invisible?
a) would become b) became c) were become
102. If he ………………… to İstanbul, he......................the Blue Mosque.
a) would go / visited b) went / would visit c) were go / visited
103- She...................all over the world if she.........................rich.
a) would travel / would be b) travelled / would be c) would travel / were
104- If I.........................his number, I...........................him.
a) had / would call b) would have / called c) would had / would call
105- If I...............................you, I.......................that man.
a) were / helped b) weren't / don't help c) were / would help
106- If we........................a yacht, we........................the seven seas.
a) have / would sailed b) had / would sail c) would have / sailed
107 - If he...........................more time, he.......................karate.
a) will have / learns b) had / would learn c) has / would learn
108. Change the statement to passive. “I was making a plan.”
a) A plan was made by me b) A plan was being made by me. c) A plan is being made by me.
109. Change the statement to passive. “She looks after the child”.
a) The child is looked after by her. b) The child were looked after by her. c) The child are looked after by her.

110. Change the statement to passive. “She had cooked some dishes.”
a) Some dishes have been cooked by her. b) Some dishes were cooked by her. c) Some dishes had been cooked by her.
B) Complete the sentences with the correct forms of the verbs in parentheses.
1. If a memory-erasing pill ……………….. (exist), I...................................(not take) it.
2. If doctors.................(turn) on the memories of people with Alzheimer’s disease, those
people.........................(get) their lives back.
3. You ……………….. (not be) able to protect yourself from danger if.....................(delete) all
your bad memories.
4. If scientists …………….. (be) able to change people’s memories, criminals.....................(try) to
give people fake memories.
5. You ………………… (lose) a good memory if a doctor...................(make) a mistake.
6. Is it.............................(possibility)to improve your memory?
7. Having a good memory is important to be.......................(success) at school.
8. I study hard, but I keep forgetting.........................(importance) information.
9. Playing games is an..................................(ease) way to improve your memory.
10. Today’s exam is more................................(importance) than yesterday’s exam.
11. They did it in order...............more about the personalities of the video gamers. (learn)
12. In order...................the device, he keeps it in a special case. (not damage)
13. This group was set up......................teenagers who may be addicted to gaming. (help)
14. In order....................her parents, she decided not to discuss the problem with them. (not upset)
15. I’m joining in.....................................my French. (not forget).
16. Cyberbullying............................................(make) victims feel they are unable to escape bullying.
17. Many students.............................(ensure) their social media accounts are set to private.
18. Amira always.........................(read) her comments carefully before she posts (post) them.
19. I only.......................(spend) time online when I have finished my school work.
20. Teachers of older children usually.............................(remind) students to keep their personal
information private.
21. Paul..........................................................(send) to prison. (Future tense)
22. My brother..............................................(just beat) in the race. (Present perfect)
23. He.........................................(tell) to wait outside. (Past tense)
24. I..............................................(not pay) for the work. (Past tense)
25. Policemen.....................................(often ask) the way. (Present tense)
26. People with PTSD …………………. (have) happier lives if doctors...................(remove) their worst
memories.
27. If I ……………………. (be) a millionaire, I..........................(live) in London.
28. She ……………….. (be) going to the beach this weekend if she (not be)....................working.
29. I …………………. (move) to Canada if I.........................(speak) English better.
30. If I ………………….. (be) working there, I............................(quit).
31- Hana is the most.............................(creation) student in our school.
32- The exam paper was so...........................(shock) for her.
33- Her dress was so.......................................(style)
34- The movie I watched yesterday was......................................(boredom)
35- He had.............................(electricity) shock last Friday.
36. They had a lot of participants in order...............that gaming attracts different types of people. (show)
37. I read it twice in order.............................any important information. (not miss)
38. In order...................the privacy of the participants, the team didn’t publish their names. (protect)
39. She always..........................(remember) her password so she doesn’t have to write it down.
40. Social media platforms have (have) an age limit so that younger children..................................(be /
not) on them.
41. He always...................................................(create) very strong passwords.
42. Our teacher always..............................(give) us lots of homework.
43. We........................................(not want) to go to the concert.
44. Their lawn...........................................(cut) once a week. (Present tense)
45. We will........................................(ask) by the police. (Future tense)
46. The towels was........................................(not use). (Past tense)
47. The jewels........................................(not sell) so far. (Present perfect)
48 They......................................(teach) French. (Present tense)
49. My sister and I hate old black and white films. (hate)
50. She.............................as a travel writer. (work)
51. My dad always.........................breakfast before us. (have)
52. They.....................taekwondo on Tuesdays. (do)
53- If I...............(have) the latest virtual reality headset, I might explore distant planets from the
comfort of my home.
54- If she owned a quantum computer, she could............(solve) complex problems in seconds.
55- If I had a teleportation device, I would....................(travel) to any destination on Earth.
56- If there was more funding available for research, scientists would................(make)
groundbreaking discoveries in various fields.
57- If they ……………….. (design) a universal translator, they might understand and …………………….
(communication) with people from any culture effortlessly.
58- The poison in this snake can be fatal. Fortunately, most snakes aren’t....................(poison).
59- It takes skill to be successful at my job, so I hope to become a.....................(skill) employee after I
gain some experience.
60- We had to raise our voice to be heard above the noise in the office. It’s hard to get any work done
in a....................(noise) workplace.
61- A police officer is used to facing danger. Fighting crime is a.....................(danger) profession.
62- My boss and I had a long talk at the meeting. She’s a very..........................(talk) person.
Part Three: Writing
1- Write a balanced opinion essay on using technology to record dreams. Write at least 250
words showing your opinion.

2- ‘Is it good to sleep for a long time?’ Write an opinion essay of at least 250 words
showing your opinion.

3- ‘Replacing fossil fuel cars with electric cars’. Write an opinion essay of 250 words
showing your opinion.

4- Your friend, Sara, has written you a letter asking for advice about how to start a new
company in Bahrain. Write a letter in which you tell him/her about the steps he/she has
to follow. (200-250 words)

5- Your friend, Ahmed has written you a letter asking for advice about how to start a new
travel agency in Bahrain. Write a letter in which you tell him/her about the steps he/she has
to follow. (200-250 words)

6- Your friend, Abdullah, has written you a letter asking for advice about how to start a new
supermarket in Bahrain. Write a letter in which you tell him/her about the steps he/she has
to follow. (200-250 words)

7- Your friend, Mohamed Ali, has written you a letter asking for advice about how to start a
new factory in Bahrain. Write a letter in which you tell him/her about the steps he/she has to
follow. (200-250 words)

8- Your teacher has asked you to write a report about the benefits and drawbacks of e-books.
Write a balanced report showing the pros and cons of the topic. Your name is Noor Ali
(180-200 words)

9- Your teacher of English has asked you to write a report about the advantages and
disadvantages of social media. Write a balanced report showing the pros and cons of the
topic. Your name is Reda Saad (180-200 words)

10- Your teacher has asked you to write a report about the benefits and drawbacks of Online
shopping. Write a balanced report showing the pros and cons of the topic. Your name
is Diaa Adel (180-200 words)
11- You are a reporter for a local newspaper. Write an article about the internet and its
positive impact on people. (200-250 words)

12- You are a reporter for a local newspaper. Write an article about the negative impact of
eating junk food on people. (200-250 words).

13- You are a reporter for a local newspaper. Write an article about mobile phone and the
positive impact on teenagers. (200-250 words)

14- Bahrain is a country full of a variety of tasty local dishes. Write a blog post describing TWO
local dishes to express the traditional food of Bahrain. Describe them and say what you like
about them and why. (180-200 words)

15- Bahrain is a country full of a variety of traditional costumes. Write a blog post describing
TWO traditional costumes to express the traditional clothes of Bahrain. Describe them and
say what you like about them and why. (180-200 words)

16- Bahrain is a country full of a variety of significant cultural and historical places. Write a
blog post describing TWO iconic places of Bahrain culture. Describe them and say what
you like about them and why.

17- Bahrain is a country full of a variety of significant cultural and historical events. Write a
blog post describing TWO significant events of Bahrain culture. Describe them and say
what you like about them and why.

18- Bahrain is a country full of a variety of significant cultural and historical symbols. Write a
blog post describing TWO significant symbols of Bahrain culture. Describe them and say
what you like about them and why.
Writing Topics
1- Writing an opinion Essay

Sleeping while Driving?

Introduction Everyone is so busy these days, and not getting enough


- Includes a thesis statement, sleep is a real problem. One serious effect of this is falling
which introduces the main idea
of the essay. asleep while driving. Self-driving cars might be a solution to
- Start with a general idea and this danger in the future. Car manufacturers think that in the
become more specific. Then
future, we will sleep comfortably in our self-driving cars.
present the thesis statement
with the main idea or your However, having self-driving cars on the road could have both
stance on the issue. advantages and disadvantages.

Paragraph (1) On one hand, sleeping in our self-driving cars could make us
- State the main idea of the safer and healthier. Tiredness is one of the major causes of car
essay.
- The first supporting
accidents. Self-driving cars will take control when a driver is too
paragraph should focus on a tired. This will probably mean fewer deaths on the road. In
single point or idea. addition, some people think that self-driving cars will help people
- Start each one with a clear get more sleep. Over a third of us do not get enough sleep. This
topic sentence. Then give facts,
causes health problems. People with long commutes might get
examples and/or details to
support it. an hour or two extra sleep while their cars drive them to work.

On the other hand, self-driving cars might not be reliable. First, like any
Paragraph (2)
- Gives supporting information about the main idea. The paragraphincludesatopic sentence,supportingideas, and details

Conclusion
- Includes a summary
statement that restates the
thesis statement using
different words and / or a
final thought about the topic.
In conclusion, there are pros and cons for people
sleeping in self-driving cars. They might help people who do
not get enough sleep. Unfortunately, though, there are also
serious safety concerns. Before you try a self-driving car, it is
important to consider the advantages and disadvantages.
2- Writing a letter about how to start a business in Bahrain
3
Greetings
Hey / Hi! / Dear…., Hey Noor,
I hope you’re doing well! It’s awesome to hear that you’re
Introduction thinking about starting a creative business in Bahrain. I’d love
- React to your friend’s to share some tips to help you get going in the right
letter and write the reason direction.
for writing.
First things first, you’ve got to figure out a creative business
Body paragraphs idea. Do some research to identify your target audience,
competitors, and potential gaps in the market. Understanding
Paragraph (1)
the local market will be crucial for success.
Step 1/ tip1
Topic sentence 1: Next, you’ll need to put together a solid business plan. This
Supporting ideas should outline your goals, financial projections, marketing
strategies, and how you plan to run things day-to-day. A good
Paragraph (2)
business plan will be your roadmap. Personally, I recommend
Step 2/ tip2
contacting the Economic Development Board (EDB) for more
Topic sentence 1:
guidance.
Supporting ideas
After that, you’ll have to register your business with the
Paragraph (3) Ministry of Industry, Commerce, and Tourism in Bahrain.
Step 3/ tip3 There are forms to fill out and fees to pay. In fact, it’s a good
Topic sentence 1: idea to get some professional help for this part, too.
Supporting ideas
Finally, always keep in mind the value of networking. Join
Paragraph (4) local business groups and events to meet fellow
Step 4/ tip4 entrepreneurs and potential clients. Collaboration with other
Topic sentence 1: creative minds can be a game-changer.
Supporting ideas In short, never forget that starting a business is a journey, and
there may be challenges along the way. Just stay persistent
and flexible. Learn from your experiences, and you’ll do great!
Concluding paragraph Wishing you the best of luck with your creative business in
- Closing tips / Reassuring Bahrain!
phrases / Saying goodbye /
Ask for a response Take care,
Wesam
3- Writing a report

Heading To: ……………………………………


To: …………………..
From: …...............
From: ……………………………….
Date: ……………….. Date: ………………………………..
Subject: …………… Subject: ……………………………
Introduction Introduction
- Briefly introduce the topic Video gaming is a popular pastime among our students, with
of the report. many engaging in this form of entertainment on a daily basis.
- State the purpose of the
The aim of this report is to examine the positive and negative
report.
- Indicate the sources of aspects of playing video games. The information included
information used for the herein was collected from different sources, including students
report. and the internet.
Pros Pros
Topic sentence: ……………… Video games have many benefits. Above all, they can
- Benefit 1: Provide details enhance students’ skills, including critical thinking and
and examples
creativity. Gaming can also relieve stress. Some students said
- Benefit 2: Provide details
and examples: that it offers them an escape from daily pressures. Moreover,
- Benefit 3: Provide details many video games encourage multiplayer modes. These
and examples encourage social interaction and teamwork.
Cons Cons
Topic sentence: ……………… Excessive gaming can lead to many problems. To start with, it
- Disadvantage 1: Provide can cause health issues like eye strain and sleep disorders.
details and examples Besides, some students reported that they are addicted to
- Disadvantage 2: Provide some games. According to research, addiction leads to social
details and examples:
isolation as they spend less time with friends and family. Worst
- Disadvantage 3: Provide
details and examples of all, some students believe that playing video games has
affected their grades. A few stated that although their grades
Conclusion
- Summarize the main findings are good, they no longer enjoy going to school.
of the report Conclusion
- Stress the complexity of the In summary, playing video games among our students is a
issue and the need for urgent
action: complex topic. Despite the numerous benefits, some students
- Make some recommendations seem to be badly affected. The information gathered for this
(at least one) report reveals that they need urgent help. I believe that our
school has to take immediate action to support them.
4- Writing an article

Online communities are not for the young


Introduction
- Background information about Young people often want to join virtual communities
the topic. because their friends are members. However, there is
- states the main idea of
increasing evidence that online communities can affect
the newspaper in a thesis
statement which explains the children and teenagers negatively.
purpose of the article.

Recent studies have shown that using social media a lot can
Body paragraph (1)
cause emotional problems. People that spend a lot of time in
Topic sentence:
online communities are more likely to feel worried and unhappy.
reason for your opinion
Part of the problem is a feeling called FOMO, or “fear of missing
supporting ideas
out.” Many people post stories about exciting things in their
evidence or examples to
lives. When someone else reads these posts, they may feel their
support the reason
own life is not exciting enough. This is especially true for young
people.

Body paragraph (2) It is well-known that young people spend a lot of time looking
Topic sentence: at social media on their phones and tablets. According to a
reason for your opinion recent documentary about this topic, looking at screens too
supporting ideas much can make it harder for people to sleep. The documentary
evidence or examples to explained that young people, in particular, need plenty of
support the reason sleep. One expert in the film said that not sleeping enough can
cause serious health problems, including stress.

Body paragraph (3) Additionally, spending too much time online can have a
Topic sentence: negative impact on young people’s education. Recent articles
reason for your opinion
make an important point about online communities: they
supporting ideas
evidence or examples to suggest that children who don’t study enough often find it hard
support the reason to be successful in the future.

Conclusion In conclusion, virtual communities are good in some ways, but


- It summarizes your main they can affect young people’s mental and physical health, as
idea and opinion. well as their future success. Parents and other adults should
- It may also include a final limit how much time children and teenagers spend in these
thought about the topic.
communities.
5- Write a personal blog post

Title EXPLORING SCOTLAND’S MAGIC

Introduction When you think of Scotland, you think of all the beautiful
- Setting the context things that make it special. These include art, stories, and
(Scotland’s most beautiful delicious food, but what really stands out to me are the
features) kilts and bagpipes. Let’s take a closer look at why these
- Identification of two two
unique symbols
things are so important in showing what Scotland is all
about.
Body Paragraph 1 Let’s talk about kilts first. They’re cool skirts made from
- The first symbol that
warm wool with colorful lines called tartan. Back in the old
reflects Scotland’s spirit
and history. days, only men wore them, but now even women wear
- Describe the symbol. them. You can spot them at parties, weddings, and just on
regular days. They’re like a symbol of Scotland’s strong
- Why this symbol
is important. spirit and history.

Body Paragraph 2 Then, there are bagpipes. These unique musical


- The second symbol that
instruments proudly represent Scotland. They’ve got a bag
represents Scotland.
and some pipes. Someone blows into the bag, and the air
- Describe the symbol. comes out of the pipes, making this incredible sound that
sends shivers down your spine. They’ve been around for a
- Why this symbol long time. The Scottish people play them at all sorts of
is important. events, from happy weddings to sad funerals.

Conclusion
- Your final comment. For me, kilts and bagpipes are inseparable symbols of
Scottish heritage. Their ability to preserve Scotland’s
- Emphasize the role of
these two symbols in identity is truly remarkable. They are old cultural
preserving Scotland’s treasures that you can touch and hear.
cultural identity.
Part Four: Listening
Q1- Listen to an interview with a woman called Jo Baylis, who is a radio presenter and
choose the correct answer a, b or c.
1 Jo decided to become a radio presenter because …….
A. she was excited about an experience her father had.
B. she believed that she had the right personality for it.
C. she loved a particular radio programme.
2 What does Jo say about her first experience of being on the radio?
A. she had little time to prepare before her first show.
B. she was disappointed by the lack of training offered.
C. she was too nervous to perform well.
3 Jo prefers to present programmes which focus on……
A. discussing different topics.
B. introducing new music.
C. reviewing films and TV programmes.
4 What does Jo say about having to get up early in the morning to present her show?
A. Jo needs time to get herself into the right mood.
B. Jo has overcome her initial problems with it.
C. Jo regrets the effect it has on her social life.
5 What does Jo say about the comments she gets from her listeners?
A. Jo tries to respond to all the points people make.
B. Jo wishes people would try to be more positive.
C. Jo is surprised by the range of opinions people have
Q2- Listen to a radio program about “Racing World” and decide whether the following statements
are true (T) or false (F)
The sentence True False
1 The first Dakar race began in Paris.
2 There were fewer competitors in 1982 race than in 1978.
3 Crossing the Ténéré desert was an exciting & easy experience.
4 1988 was the tenth anniversary of the race.
5 The 22nd Dakar started in Senegal and ended in Egypt.
Q3- You will hear people talking in five different situations. For questions, 1-5, choose the
best answer A, B or C.
1. You hear two teenagers talking about a lost scarf.
Where does the girl think she lost it?
A) on the bus B) in the street C) in a shop
2. You hear two people talking about a proposed new motorway.
What is the man worried about?
A) the noise from fast-moving traffic
B) the loss of local sports facilities
C) the permanent harm to the countryside
3. You hear a radio announcer giving a traffic update.
Who is his advice for?
A) People going shopping.
B) People going to watch football.
C) People going to a music festival.
4. You overhear a woman talking on the phone.
What does she want the other person to do?
A) to meet her somewhere
B) to give someone else a message
C) to buy something for her
5. You hear two people talking about holidays.
What is the woman encouraging the man to do?
A) Visit particular countries
B) go on an environmentally-friendly holiday
C) travel with a large group of people
Q4- You are going to hear Simon talking about how he learned several languages in a variety
of different ways, then decide if these statements are true ( T ) or false ( F )
The sentence True False
1. Simon lived in Europe, Middle East and South America
2. Simon learned five different languages.
3. Before moving to Hong Kong, he lived in Barcelona for 10 years.
4. He learned Italian by attending classes.
5. He learned Italian by Speaking to people and using a dictionary.
Q5- Listen to an interview between Iris Battle (a forensic scientist) and a researcher for the
documentary programme and choose the correct answer:
1- Iris Battle said that they work in ………………….. .
A) 5 teams B) 2 teams C) 3 teams
2- The analysts work is to ………………… :
A) Do the test in the lab B) collect ideas C) search for and collect the evidence
3- The analyst in the TV drama……………… .
A) Search and collect the evidence B) do the test in the lab C) both A and B
4- They have to wear.......................in order not to damage the evidence.
A) rubber gloves B) rubber masks C) rubber kits
5- To know where the evidence has come from, they have to ………..
A) take photos B) take fingerprints C) take notes
Q6- Listen to this talk and decide whether these sentences are true, false by putting tick
(√) in the correct place:
The sentence True False
1- The talk is about the increase in the number of university students studying
languages.
2- Students think that languages are as important as science and math.
3-Students have a negative view about learning languages because of their parents.
4- Technology gave students a negative attitude towards learning English.
5- The speaker thinks that understanding another language is the basis for
understanding another culture.
Q7- Listen to the following report talking about a Dacha Season then decide whether they
are true (✔) or false (X)
The sentence True False
1. A dacha is a holiday home.
2. People go to dachas to relax and do some gardening.
3. dacha is a large flat.
4. Many dachas are not suitable for winter.
5. The dacha season usually starts in January.
Q8- You will hear people talking about running. Listen and choose the correct answer a, b or c.
1. Tim started running when he was ……..
a. 6 years old. b. 7 years old. c. 10 years old
2. What type of races did Tim take part in?
a. Sprints. b. Middle distance. c. Long distance.
3. Tim 's fastest time in 1,500 meters is …
a. Three minutes and forty-seven seconds.
b. Three seconds and forty-seven minutes.
c. Three minutes and seventy-four seconds.
4. Tim is slowing down because he is …
a. Getting older. b. Eating too much. c. Using the car.
5. How old is Tim?
a. 72. b. 33. c. 27.
Q9-You will hear people talking about sleeping hours. Listen and choose the correct answer
a, b or c.
1) Hana usually sleeps a night.
a. six hours b. seven hours c. eight hours
2) Daniel usually sleeps Hana.
a. the same as b. less than c. more than
3) Hana gets extra sleep by .
a. taking naps every day.
b. sleeping Saturday morning.
c. going to bed early.
4) On Sundays Hana .
a. sleeps a lot. b. sees friends. c. takes a nap.
5) At the weekend Daniel .
a. gets up early. b. sleeps less. c. takes naps
Q10- Listen to the following report talking about a special drinking straw and decide
whether the following sentences are true (🗸) or false (X).
The sentence True False
1. Many people around the world do not have clean drinking water.
2. There are no water purification facilities in poor countries.
3. The straw contains special chemicals.
4. The straws are very simple to use.
5. These straws are the best way to provide clean water for everyone.
Q11- Listen to a speaker talking about an animal rescue society in Australia and choose the
correct answer a, b or c.
1- The Wildlife Information and Rescue Service Centre helps …….
A. all kinds of animals. B. only local animals. C. birds and cats.
2- The courses which are provided by the Wildlife Center are …..
A. impractical. B. rather disappointing. C. useful and educational.
3- What does John’s dad do for the organisation?
A. He tries to save animals that are hurt.
B. He answers the phone at the Wildlife Centre Office.
C. He chooses someone to go and see the animal.
4- What happened to the baby bat?
A. It was killed by other bats.
B. It is now looked after by John at home.
C. It was rescued and put back into the wild.
5- John's future plan is ….
A. taking care of all kind of bats. B. helping injured animals. C. opening a new rescue organisation.
Q12- Listen to a man called Steve and a woman called Caroline talking about summer jobs
and decide whether the following statements are true (T) or false (F).
The sentence True False
1 Steve hasn’t arranged any work for the summer yet.
2 Caroline’s work will be located in a city.
3 Caroline’s work will allow her to have free time during the day.
4 Caroline found out about the job from the internet.
5 Caroline does not have to pay for her accommodation.
Q13- Listen to the podcast between Michael and Dr. Jennifer and choose the correct answer:
1- The program introduced by Michael is called …………………… .
a) Video Games b) All Things Tech c) Psychologist
2- According to Michael, the first video games appeared in …………….. .
a) 1970s b) 1917s c) 1907s
3- Michael described video gamers of being..............because they ignore school.
a) unhealthy b) irresponsible c) anti-social
4- The ‘APA’ stands for …………………… .
a) American Psychiatric Association b) World Health Organization c) Internet Gaming Disorder
5- Jennifer thinks that video games require using............................to win.
a) problems b) critical thinking c) facts
6- Jennifer and Michael concluded that online games have ………………….. .
a) more advantages b) more disadvantages c) equal advantages and disadvantages
Q14- Listen to the consulting agency interviewing two of the company’s employees and put
(√) on the correct person

The sentence Anita Yusuf


1- Has the longest service record.
2- His/her work used to be more sociable.
3- Feels not enthusiastic at work now.
4- Used to work as a team.
5- Does not think he/she can achieve more work.
6- The company helps him/her to develop skills.
7- The company pays him/her more than in the past.
8- Has no problem about not travelling in the company.
Q15- Listen to part of a radio programme about a new recycling centre, and then decide
whether the following statements are (True) or (False).
The sentence True False
1. The centre is located outside the town.
2. Recycling at the centre is always free.
3. All people in Dunham liked this environmentally-friendly centre.
4. Shelly thinks that the centre is far away from the town.
5. Shelly believes that the town mayor cares about the environment.
Q16- You are going to listen to five speakers talking about sports. Match statements to the
speakers (1-5). Note that there is one extra statement.
Statement Speaker
A. I was injured when playing my sport.
B. I enjoy doing my sport alone.
C. I prefer playing sports I can do in a team with other people.
D. I train people to skydive.
E. I usually practise with my family members.
F. I am always looking for a new sport to try.
Q17- Listen and complete the notes in the table: (ONE WORD only in each gap)
Cons of playing video games Pros of playing video game
some gamers are unhealthy games can develop (5)..........................skills
• don’t (1) ……………………… • find (6)...................to problems
• eat junk food • make (7)......................decisions
some are anti-social • follow instructions
• no (2) …………………. • develop (8) …………………..
some ignore important things better (9)...................skills
• school, work, (3) ………………………… • strong sense of (10) ……………….
some are (4)....................but a trivial number • more (11) ……………………
• (12)......................loneliness
Q18- Listen to a report about "Sign Language" then mark the following statements True (T) or
False (F).

The sentence True False


1. Sign Language was invented in the 19th century.
2. Abbe Charles Michle L'Epee created a school for who hear.
3. Sign languages have different grammar but similar vocabulary to spoken language.
4. Deaf people in South Africa speak 11 different sign languages.
5. Gestuno is a sign language that is used at the Olympic for the deaf.
Q19- Listen to Bruno talking about designing video games, then choose the correct answer:
1. Bruno Muller has been a game designer for …………………………. .
a) exactly 10 years b) less than 10 years c) more than 10 years
2. Bruno thinks game design is a good career choice for people who are ……………….
a) creative b) positive c) fun
3. Bruno likes the fact that the designers wear ……………………… .
a) new clothes to work. b) formal clothes to work. c) casual clothes to work
4. Bruno points out that some projects can ………………….
a) deal with b) just fail c) give up
5. Bruno says game design is a great career for people who ……………………… .
a) don’t get upset easily. b) don’t like to work many hours. c) don’t have a lot of patience. Q20-
Listen to the conversation between a customer and a salesperson; and complete the table
below. (ONE TO THREE WORDS)
Device ………………………….
Problem a. ……………………………. is slow b.........................isn’t good
Method of payment cash or ………………………
The customer’s last name ……………………..
The customer prefers to exchange or ……………………..
Q21- Listen to this dialogue between the manager and Diane and complete the booking form
below.

Tower Bridge Restaurant


Day: ………………………….. Time: …………………..
Name: …………..……………….
Number of people: ……………….
Number of vegetarians:
Q22- Listen to the interview and make (T) for true, (F) for false or (NG) for not given:
The sentence T, F or NG
1 The speaker lives in New York.
2 His parents’ house is made of stone.
3 Life in Manhattan is busier than Staten Island.
4 The speaker’s grandparents live in the U.S.
5 The speaker’s cousins have their own business.
6 People grow apples in Bolzano.
Q23- Listen to the podcast. Choose the correct answer
1. Why is Angela a guest on this podcast?
a) because she spent a year studying abroad in Norway.
b) to give travel advice about the Lofoten Islands.
2. Why did Angela want to go to Lofoten?
a) because it’s far from Vancouver. b) because of her grandfather.
3. Why did Angela choose this study abroad program?
a) because her parents preferred that program.
b) because she felt she could depend on the program’s staff
4. Why was saying goodbye at the end of her stay especially difficult for Angela?
a) because she would miss her host sister. b) because she would miss the great food.
5. Why did Angela take so many photos?
a) because the Lofoten Islands are so beautiful.
b) because she has a special interest in photography.
6. Why was it important to Angela to have the right clothing?
a) because she cares about presenting the right look.
b) because the weather in Lofoten changes often.
7. Why did Lofoten remind Angela of Vancouver?
a) because of the mountains and the sea. b) because the streets were so full of people
Q24- Match each speaker with his/her dream house:
Dream house Speaker
a) wants a house away from city and noise.
b) wants a house near the sea with many windows.
c) wants to have a house with high ceiling and Jacuzzi.
d) wants a very big house with a cinema.
e) wants a small apartment in a lively city.
Q25- Listen to a report about London’s Top Historical Attractions and complete the table
below:
The Tower of London The Tower of London is the oldest (1) …………………
(2)............................and prison in the Europe.
The Houses of Parliament The palace was offered to the government in (3)
……………..

The palace was destroyed by (4).....................n 1834


Madam Tussauds The small wax museum in London had (5) ……………….
wax figures.
Q26- Listen to the funding committee discuss the project and fill in the gap with the
suitable word or number:

Q27- Listen to the lecture and choose the correct answer:


1- Name of the project: …………………..
1. What is the speaker’s main point about the evolution, or development, of languages?
Languages
a. 2- influence oneareas
Urban wasteland another.
make the place feel.......................and dirty.
b. Changes are happening faster today.
3- The
c. We project
should try to needs.................Dollars
prevent change. a year.
2. What does the speaker say about Arabic?
4- They need money for paying to people who will look after the ……………..
a. It borrows many English words.
b. 5-
It isThey
known for its
offer colorful
5000 expressions.
dollars per year for...................years.
c. It had a strong influence on Spanish.
3. What do you call a version of a language that’s used by a particular group?
a. an expression b. a dialect c. a case of borrowing
4. What does the speaker say about the words software and tweet?
a. They have replaced other words.
b. They are loanwords related to technology.
c. They mean different things in different languages.
5. What’s an example of a loanword that changed its meaning when it was borrowed?
a. handy as used in German b. déjà vu as used in English c. tweet as used in French
Q28- Listen to the description of Bill Gate's house and decide whether the following are
(T) or (F):
The sentence True False
1.The house is next to a lake.
2.There are 150 chairs in the small living room.
3. There are ten for visitors.
4.Bill Gates and his family watch films in the living room.
5.There's a private pool at the house.
Q29- Listen to the dialogue between Mary and a car dealer and circle the correct answer
1-Mary is looking for a........................car.
a) small. b) cheap. c) comfortable.
2-Lexus 430 can go at a speed of.................kilometers an hour.
a) 218 b) 208 c) 280
3- Lexus 430 costs B.D..........
a) 23450 b) 24350 c) 34450
4- Mary prefers the.................color.
a) blue. b) green. c) black.
5- The dealer gave Mary:
a) a good discount. b) a good discount and guarantee. c) Two years guarantee.

You might also like